[Previous] Asking Good Questions | Home | [Next] David Deutsch's Tweets Suck

Scheduling and Thinking

I usually do philosophy first when I wake up, when I'm fresh. This works for me because I'm not sleep deprived. Lots of people have a hard time in the mornings because they are chronically sleep deprived. Getting enough sleep is important to being a good thinker. I wake up without caffeine, an alarm, a shower, or anything to wake me up. I don't keep track, but I'd guess that I average over 8 hours of sleep per day (including naps).

I don't eat until after doing philosophy. (I find I'm not hungry for at least 4 hours after waking up.) When convenient, I use eating, exercising or showering as a break after doing morning philosophy.

Happily, I do contract software work from home and can choose my own hours to work. So I can do philosophy first regardless of what time I wake up, then do software work or whatever else later on. My sleep and wake times vary significantly and sometimes I take naps. I sleep when I'm tired, not on a schedule.

I avoid scheduling many activities. And I get a lot of stuff delivered, including lots of my groceries, so I don't need to go places during business hours much. This improves my sleep and gives me the flexibility to do what I want when I want to.

If I worked a 9-5 job, I would go to sleep by 8pm so I could wake up by 4am and do philosophy in the morning before work.

If you work a regular job, and you want to be a better thinker, I recommend you try something like this. Don't wake up for work. Sleep after work and then wake up long before work. Then you can do your own thing (e.g. study critical thinking skills and read philosophy books) when you're fresh. This can also work with anything intellectual, e.g. if you're writing a novel. It's really hard to do your best thinking after an 8 hour workday.

There are some problems you may run into:

1) Missing new TV shows. You should record your shows for watching later, or buy or torrent them online. You don't have to watch TV shows when they air. If people at work will discuss the show the next day and you want to join the discussions and avoid spoilers, consider watching in the morning. E.g. do philosophy from 4-7:30am, then watch the TV show (while eating breakfast) then go to work.

2) Social life. If you care more about socializing than thinking, you'll want to be available in the evenings when our culture prefers to do social activities. You've gotta choose. You can't be a productive, great thinker; work a regular job; deal with life (load the dishwasher, go to the dentist, etc); and also keep up a conventional social life. There isn't time for all that.

3) Family time. If you need to stay up until midnight to have enough time with your kids and spouse after work, then you aren't going to have enough time for intellectual activities regardless of your sleep schedule. That's your choice. Good luck trying to squeeze in some thinking on weekends, I guess, but that's when you'll be busy going places and doing family and/or social activities that don't fit on weekdays.

4) I need to be at my best at work. If you're especially smart (like most people interested in my blog who want to be intellectually productive outside of work) then you can do most jobs just fine without being at your very sharpest. You aren't being paid extra to work right after sleeping (actually lots of people come in tired in the morning because they're sleep deprived, and that's perfectly acceptable at most jobs). If you're struggling at work, then OK focus on that but don't expect to be very productive on an intellectual side project. But don't give up your own stuff to try extra super hard at work when you're already doing fine, that'll just get you exploited more than rewarded.

Real talk: If you work full time and have a family life and a social life, you're probably sleep deprived already. You've chosen how to spend your time. On the other hand, if you want to do some kind of thinking or creative pursuit, and you're willing to allocate time for it, then try changing your sleep cycle so you can do it in the morning before work, while fresh, instead of trying to do it after work.

All of this applies if you go to school rather than work. And if you don't work, then try getting enough sleep and doing your most intellectually demanding activity first thing after waking, when you're the freshest.


Elliot Temple on December 30, 2016

Messages (173)

Sleep schedule

I agree @ arranging life to not be sleep deprived, and about getting 8+ hours per night. And I've found I need it to be about the same time each day, or else I have problems with things like failing to fall asleep when I want, or waking too early and not being able to go back to sleep. I've tried different sleeping approaches and feel "stuck" in the one I currently have.

I have what would be best approximated as a 6am - 6pm+ job schedule, but fortunately with a decent amount of available break time during the day. I'm not "heads down" the whole time and on a typical day I get several 1/2 - 1 hour breaks. But the breaks are fairly unpredictable as my schedule tends to be dominated by a combination of pre-scheduled group conference calls and impromptu interactive sessions of various types.

The 6am job start time is itself a balance of forces between the demands of the job and my wakeup time. There is fairly constant pressure for me to start job stuff even earlier, especially in the summer ("Can you please take a call at 5am your time?" is something I get asked a lot) The pressure comes because nearly everyone I work with lives in time zones ahead of me, almost all are 3 or more hours ahead in summer, lots of people in Europe, and the most difficult is a group in Pakistan that's 12 hours ahead. So a meeting at 6am my time is 6pm their time - cutting into their dinner and family time. Or vice-versa (6am for them and 6pm for me). I also work with a few people in Japan, New Zealand, and Australia who are "closer" to my time zone meeting late in my day and early their next day. All these groups meet with me, and also others, also scattered all over the world which is one of the reasons my job doesn't usually end until fairly late in the day despite starting early and having mid-day breaks.

I have made a point of trying to hold my 6am start time "firm", and with 1-2 exceptions per year I am successful at doing that. But if I was routinely up earlier (say 5am or 4am) I would have much less resolve about not starting work activities until 6am, as I'm already regularly turning down invites to 4am or 5am calls and causing scheduling difficulties with team mates because of it.

The reason I'm not routinely up earlier boils down to my sleep start time, which is typically 9:30 pm. This owes to meal times + the problem you listed as #3 "Family Time". Even 9:30pm was, itself, quite a negotiation to achieve for reasons I won't go into. But a significantly earlier bed time would require a major rework of how my family functions. That's not out of the question, but it's not a simple undertaking either.

I've tried some other things to address sleeping and scheduling. One was shorter night sleeps with mid-day naps. In theory I should be able to do something like 5 hours sleep at night + 3 in the afternoon. This generally didn't work for me because, say I plan a nap 1pm - 4pm. Well, everyone at work expects you to be awake and available between 1pm - 4pm your time, even if you fairly rarely have actual scheduled calls during that time. But people will randomly schedule stuff there, or call my work line, and if I'm not available some get pissy and "nap" isn't a socially acceptable excuse for routinely being unavailable during what's considered "prime working hours". Another problem with naps is people outside and inside the house make noise, which even ear plugs doesn't fully filter out and wakes me up.

I've also tried physically shifting my location so that I'm in a different time zone. The summer before last I spent a good portion of on the US East coast, which pushed all the work schedule 3 hours later local time. That actually worked quite well from a time zone / sleep schedule perspective. Made my start time 9am like a "regular" job, and people didn't ask for as many "oddball" stuff because I was part of a mass of people in the US Eastern time zone so other people would more readily shift to accommodate. But long term I don't wanna live on the US East coast just for the time zone and neither does my family.


AnonITA at 8:17 PM on December 30, 2016 | #8131 | reply | quote

i don't see an easy fix here given the 12 hour work day. (i understand the breaks help with many things, but not with this). with an 8 hour work day it'd be far easier to have 2-4 hours before work!

waking up 1-2 hours before work and being *selfish* about it, and being unavailable, is worth considering though. screw their pressure. just don't tell them you're awake. if you want to do this, just do it (privately), don't let their pressure mess it up. if you get caught occasionally you could just say it was an exception that you woke up early that day. you might give in on early calls more often as a result, but if you were regularly waking up at 4 or 5 then that wouldn't be a big deal. and if you hold the line most of the time, then you have 1-2 morning hours to yourself most of the time. even if you only kept it to yourself half the time, it'd still work a lot. but it's hard to hold the line 50% of the time, so you better hold the line more like 90%+ of the time. if you made one weekday exception per 2 weeks then it could still be seen as a special exception. however this run into family time some to get enough sleep. the basic plan of the blog post really doesn't work with extra 4 hours of work day. oh well. (it will also be a tight fit for people with longer commutes who don't live alone.)

i think flexible sleep times are difficult unless your schedule is flexible most of the time and has few commitments at specific times. with anything like a normal schedule, sleeping consistently makes reasonable sense. it sounds like your breaks during working hours need to be quickly interruptible. this allows a lot of good activities, but doesn't work well with some others including naps.


curi at 8:34 PM on December 30, 2016 | #8132 | reply | quote

If I was going to get up at 4am I'd need to plan bed time at 7:30pm allowing for get ready / fall asleep time and wake up time. One of the nap problems would still apply - People outside think it's fine to make noise until about 9-10pm. They call & ring doorbells etc. also.

In addition we have dinner at about 6:30 or so - which is itself a compromise. Particularly in the summer, family prefers not to have dinner until around sunset. I find going to bed shortly after a big meal causes digestion problems and sleep problems. But if I don't have a big meal at night - say I have a big lunch and small dinner - I wake up in the middle of the night hungry. But then if I eat, typically can't go back to sleep, and gets my whole rhythm mucked up.

Through experimentation I've settled on 3 hours between a big dinner and bed time as a safe span. That allows me to sleep the whole night.

I could have my own dinner earlier, say @4:30pm. But then I'd have to do the dinner cooking & I don't currently have time & skills to do that all the time. And dinner is part of "family time" too.


AnonITA at 8:10 AM on December 31, 2016 | #8133 | reply | quote

The real issue

It's interesting to talk about scheduling stuff - maybe I can find some optimization somewhere.

But I don't think my schedule is the real issue. The last 2 weeks I haven't had work, haven't had conf calls etc.

I could have easily done 4 hours of philosophy right after waking if I'd wanted to.

I didn't.

These last 2 weeks I have read everything new (FI posts, blog posts & comments, etc.) right when I got up, which is different from normal.

But I rarely had anything to say, or if I did, it came to me later in the day.

And I didn't want to read philosophy books, or go back to old posts in my queue.

I just found other stuff to do once I was done reading all the new stuff. Which is also why I think if I did get up earlier when working, I'd just end up checking my work email and accepting earlier meetings, rather than doing much philosophy.

I could easily "make myself" do philosophy stuff at the appointed time, but I've been trying to avoid the "make myself" model of action.

If I was highly motivated to solve it, I think I could solve the scheduling to favor philosophy. I think the trouble is really that I'm not highly motivated.


AnonITA at 8:10 AM on December 31, 2016 | #8134 | reply | quote

the morning scheduling idea was only intended for stuff you already want to do and will do. i agree it doesn't address mixed feelings about doing some activity in the first place.

never wanting to go back to stuff in a "queue" is pretty typical for a wide variety of stuff. people often put stuff in a queue because they don't want to do it now. and if they don't want to do it now, they often don't want to do it later. this is especially typical for long term queues.

do you think that doing better quality thinking wouldn't be valuable to you, or that you don't know how to approach and achieve it, or something else?

---

requiring not just a meal but a larger meal 3 hours before bed to not wake up hungry seems pretty fragile. so do some of the other things.

i take it you eat breakfast too. i routinely go over 16 hours without eating. e.g. 8pm large meal with no snack later on wouldn't be unusual. then i rarely have breakfast or a morning snack, might eat at 2pm the next day. sometimes i go like 22 hours without eating. sometimes i do 1 meal instead of 2 in a day. i never do 0 meal days though (which mean like 32+ hours without eating, not 24, btw).

family dinner has a variety of downsides like that it's a pretty inefficient way to hold conversations (more than 2 people present with different interests and news, eating interrupting talking) and it can interrupt whatever people were doing. it teaches everyone not to get involved in hard-to-interrupt activities. it makes people limit those from their lives. or do the same activities in less intensive ways. e.g. it's possible to do philosophy writing and always be interruptible. but that's costly. it's better if sometimes you're really into it and want to finish something and don't want to be interrupted by dinner.

lots of foods reheat well. so you or others could get a reheated version, rather than everyone eating at different times.

there are also options like warming drawers, sous vide (steaks can be cooked like 1-4 hours, there is a large time window to stop cooking yours), and foods where you do the prep and then there's a short cooking phase that can do individual portions whenever.

and you don't have to respect cultural meal categories. you could e.g. reheat some dinner food for breakfast the next day. then each a lunch food you cook yourself for dinner.

it saves substantial cooking effort to frequently making larger portions and save some for later reheating instead of cooking daily. and it gives people more options. just eating the one meal of the day kinda sucks. people get used to it and don't realize it's a problem. but if they had 20 options every day at no cost, they'd learn that choosing what you want is nice. variety is valuable enough to keep lots of non-perishable foods around and sometimes eat them even if they are lower quality food. sometimes i'd rather have a canned soup than a fresh well-cooked chicken breast. non-perishables also generally make great snack options and require low cooking effort. but you can also have some recent leftovers that reheat well providing more options in both the fridge and freezer.

some of the foods that reheat badly can be adjusted to reheat well. for example, i frequently microwave subway sandwiches. i actually prefer them microwaved rather than fresh from the store because i like them hotter than subway will make them. this may sound gross. there's a reason for that. lettuce sucks in the microwave. so i get my sandwiches with no lettuce. lettuce is bland and wasn't a priority topping for me anyway. (if i'm eating a sandwich immediately with no microwaving, i might get lettuce, or still might not.)

it's pretty convenient to have some foods like mcdonalds burgers (the varieties with no lettuce), taco bell burritos (general rule of thumb is tacos have lettuce, burritos don't), or subway sandwiches in the fridge for some great options to reheat. it you put them in tupperware (or spin the opening of the subway plastic bag to seal it, then set the sandwich on top of the opening) then they will last over a week in the fridge. burger buns dry out too fast if you don't use tupperware or a plastic bag or something. people don't do this because it's not how our culture thinks of food. plus the largest beneficiaries would be children who more often love these foods. and food from more "fancy" restaurants generally spoils faster and/or reheats less well. e.g. if you get extra mexican food from other places you'll find that besides lettuce, sour cream and guacamole both microwave badly. taco bell however has a special version of sour cream that does fine in the microwave! i assume in some way it isn't real sour cream. it tastes good though. pizza is another food that's easy to buy extra of from restaurants and reheats very well.

a lot of foods reheat better with a little skill, too. like reheat with the microwave on a low power setting for a long time. or cut things in half or smaller pieces before reheating. (this often violates some cultural food presentation norms, but is harmless to taste). technology is helpful here too (inverter microwave i think it is. the thing that lets it actually do low power for real instead of just alternating full power and no power to fake lower power cooking).


curi at 12:10 PM on December 31, 2016 | #8135 | reply | quote

> i take it you eat breakfast too. i routinely go over 16 hours without eating. e.g. 8pm large meal with no snack later on wouldn't be unusual. then i rarely have breakfast or a morning snack, might eat at 2pm the next day. sometimes i go like 22 hours without eating. sometimes i do 1 meal instead of 2 in a day. i never do 0 meal days though (which mean like 32+ hours without eating, not 24, btw).

Is it healthy to not eat for over 16 hours?

Do you ignore hunger in those 16 hours?

Should I do what you are doing?

If the food science articles are crap. How do I live healthier?

I have been following your " Eating only when hungry " advise. Eating is more fun now. I play a video game or read when I am bored instead of reaching out for food.


FF at 1:39 PM on December 31, 2016 | #8136 | reply | quote

> Do you ignore hunger in those 16 hours?

no, if i'm hungry i eat. i'm rarely hungry for 4 hours after i wake up, and sometimes longer. and i often don't eat within 2-4 hours before going to sleep, and sometimes longer. especially when i have larger meals then i eat snacks less or sometimes only have one meal for the day instead of two.

note AnonITA often doesn't eat for at least 11 hours. if he has dinner 3 hours before bed and then sleeps 8 hours, that's 11 even if he has breakfast immediately.

just having a few hours before you sleep without eating, sleeping, and then not eating breakfast will get you a bunch of hours without eating. i've never thought of my eating as any kind of fast.

> Should I do what you are doing?

i don't know. i don't have anything against eating in the morning on principle. i do it occasionally.

> If the food science articles are crap. How do I live healthier?

stop worrying about living "healthier". *thinking better is way more important*. people suffer much more from bad ideas than bad eating/exercise/etc. prioritize!

> I have been following your " Eating only when hungry " advise. Eating is more fun now. I play a video game or read when I am bored instead of reaching out for food.

great. only eating when hungry is for everyone!


curi at 1:54 PM on December 31, 2016 | #8137 | reply | quote

> do you think that doing better quality thinking wouldn't be valuable to you, or that you don't know how to approach and achieve it, or something else?

I ran into lots of confusion just unpacking this question and considering it, let alone answering it. I've been finding that a lot lately.

One problem I have is with the concept "valuable".

High level, where my approach to pursuing values in life goes beyond emotions, automatized habits, and cultural defaults I think my approach is: estimate the value of a thing, estimate the cost, and if value > cost do it and if cost > value don't do it.

I know, intellectually, there's lots of valuable things in the world and money is just one of them. I know that I value lots of things in life other than money.

But I don't know how to *measure* the cost or value of anything well other than with money. And that includes, ironically, the value of knowing how to do something like measure the value of things other than money. I have no clue, and no idea how to figure it out, and no motivation to figure it out either because I don't know how valuable it would or wouldn't be. It's kind of an infinite regress problem.

Maybe time itself presents another avenue for measuring what's valuable to me. Would I study philosophy for 4 hours today to save 10 hours over the next year on stuff that I don't like doing? Sounds like a good trade, but my sense is that such clear opportunities measured in time alone are rare or non-existent. Or at least, I lack the skills to spot them.

So one thing your question exposes is: I lack the tools to intellectually figure out what's valuable to me in terms of anything other than money.

And that leads to awareness of a second problem. Straighforwardly, "better quality thinking" should be capable of producing results that I could measure in terms of money even if I can't measure them in other, perhaps more important, ways. Money should at least be a start.

But even that doesn't help. Maybe a bunch of personal backstory is needed to understand why or maybe not, but producing more money is not *now* something that I find interesting or motivating. This is a relatively recent development - for most of my life money was a pretty big motivator.

As a result I can't even answer hypothetical questions like how much time & effort I'd be willing to put in to double my annual income or double my net worth. It's not zero but kinda "meh". Which I actually think is fine - I'm not "meh" about my whole life & values, just the one thing I know how to measure pretty well (money itself) and thus have already optimized enough for my current lifestyle.

Without a way to intellectually measure what's valuable to me, I'm back to the first three ways I mentioned for determining what's valuable to me: emotion, automatized habits, and cultural defaults.

My guess is that none of these three methods tell me that better (than my current) quality thinking is valuable enough to spend major time & focused effort doing. So my guess is that's why I don't.


AnonITA at 5:07 PM on December 31, 2016 | #8138 | reply | quote

> High level, where my approach to pursuing values in life goes beyond emotions, automatized habits, and cultural defaults I think my approach is: estimate the value of a thing, estimate the cost, and if value > cost do it and if cost > value don't do it.

this is a bad calculation method because it requires everything you don't do be valued below 0, even though some of them would be pretty good activities.

the normal calculation method assigns a positive value to lots of things, then prioritizes the ones that create more value per time. this fits well with the fact of reality that there are lots of better-than-nothing activities that you don't do.

> Maybe time itself presents another avenue for measuring what's valuable to me. Would I study philosophy for 4 hours today to save 10 hours over the next year on stuff that I don't like doing? Sounds like a good trade, but my sense is that such clear opportunities measured in time alone are rare or non-existent. Or at least, I lack the skills to spot them.

there are lots of opportunities like that when you look at AVERAGE future savings. (there is substantial outcome variance.) but most of them are way too hard to accurately estimate at the outset. many are very hard to estimate after the learning phase, before the benefits phase. estimating before the learning phase is WAY harder than that because you don't really know what you'll learn or what the implications will be, especially with learning that doesn't fit into predefined buckets our culture has already established and has lots of experience with.

> And that leads to awareness of a second problem. Straighforwardly, "better quality thinking" should be capable of producing results that I could measure in terms of money even if I can't measure them in other, perhaps more important, ways. Money should at least be a start.

this is very hard to calculate. one reason: there are two different amounts of money: the amount you COULD HAVE gotten if you were maximizing money, and the amount you actually get.

my philosophy skill could definitely have gotten me millions of dollars (especially by accepting some non-monetary downsides) by now. but i've chosen to get some other things instead which i prefer, including creating some innovative philosophy ideas. more money wouldn't do a super great job of solving the problems i care about. don't you have a similar issue? i don't think you're poor. i doubt you're coming very close to maximizing money already.

so anyway if you're trying to measure outcomes by money acquired (or more accurately wealth – iMacs and houses count too, not just cash) then it's not going to work well because the amount of money you get is determined by how much you aim to get more than how much you could get. so improving your capability to get money doesn't mean you'll actually make the effort to get more money.

so i agree better quality thinking IS *capable* (your word) of producing money-measurable results (including literal money). but it's much easier to measure actualities than capabilities, so that's not super helpful.

> But even that doesn't help. Maybe a bunch of personal backstory is needed to understand why or maybe not, but producing more money is not *now* something that I find interesting or motivating. This is a relatively recent development - for most of my life money was a pretty big motivator.

i don't find that needs much explanation to make sense. i predicted some of it above. btw money has *never* been a big motivator for me. (and yet i have more money than most people.)

> As a result I can't even answer hypothetical questions like how much time & effort I'd be willing to put in to double my annual income or double my net worth. It's not zero but kinda "meh".

right.

so you should think about what problems you would value solving. this does not require measuring value. it means what are some you want to solve? what do you care about? what do you want to accomplish? what pain points do you want to fix?

you don't have to have great answers to these things. just make some guesses. ponder them for a while. discuss them. criticize. don't rush. think it over. and when some of them still seem like good ideas after a week or two, get started in some ways that don't have big upfront commitment or resource costs. if things continue to go well you may succeed, or get confident enough to pursue less tentatively.


curi at 5:29 PM on December 31, 2016 | #8139 | reply | quote

>> High level, where my approach to pursuing values in life goes beyond emotions, automatized habits, and cultural defaults I think my approach is: estimate the value of a thing, estimate the cost, and if value > cost do it and if cost > value don't do it.

>

> this is a bad calculation method because it requires everything you don't do be valued below 0, even though some of them would be pretty good activities.

>

> the normal calculation method assigns a positive value to lots of things, then prioritizes the ones that create more value per time. this fits well with the fact of reality that there are lots of better-than-nothing activities that you don't do.

Do you think your statement is true even if "cost" in my statement includes the opportunity cost of not pursing rival values you could have pursued instead? I don't think it is.

I agree with your statement if opportunity cost is not included in "cost". Then you end up with a bunch of positive values, some bigger than others, and the right thing to do is pick the ones that create more value per time.


AnonITA at 10:29 AM on January 3, 2017 | #8149 | reply | quote

Problems to solve

> so you should think about what problems you would value solving. this does not require measuring value. it means what are some you want to solve? what do you care about? what do you want to accomplish? what pain points do you want to fix?

The problem I keep coming back to is aging and death.

I think there is no problem I could not solve if I live long enough.

But unless something major that also seems entirely out of my control changes (SENS progress), I won't. The overall human lifespan seems ridiculously, woefully inadequate.

And *if I'm lucky* a significant portion of my remaining life will be spent in a relatively frail, pain-prone state at the mercy of a fairly dysfunctional health care system.

If I'm unlucky, I could at any moment die of a sudden heart attack or contract a terminal disease (both risks that rise daily as I age).


AnonITA at 10:47 AM on January 3, 2017 | #8150 | reply | quote

say you have 4 incompatible options with the following values before factoring in opportunity cost. You get to do exactly one of these:

A: 8

B: 6

D: 4

E: 2

What calculation do you want to be done for opportunity cost? Is it subtract the value of the top option (A) from all the rest since they involve not doing A? And subtract the value of the second best option from the top option?

So you end up with:

A: 2

B: -2

D: -4

E: -6

Which is misleading because doing E would create, not destroy value.

And it's misleading because the real gap between A and B is 2 points, but now it's showing as 4. (This calculation method always doubles it. It also distorts the gaps between A and the other things, e.g. the gap to E went from 6 to 8.) It also throws off the % comparisons. A is 33% better than B in the original, correct scheme of values. Now you can't even compare like that because of the negatives. And everything but the #1 option will always be negative with this method.

If this isn't how you factor in opportunity costs mathematically, you tell me.

@dying you would have a good chance to do something relevant, with a chance to help, if you focused on it (and wanted to and were interested, energetic, motivated and stuff).

one of the avenues to helping with the dying problem is to get good enough at persuasion to reach people like Aubrey de Grey who are making a lot of big mistakes. if you could get him and many others to change course -- even just a minor course adjustment -- it could significantly improve outcomes (on average -- there's still a lot of variance involved). that is one of the very important projects i've been working on and gotten very little help with.


curi at 12:10 PM on January 3, 2017 | #8153 | reply | quote

what problem does thinking about opportunity cost solve if you already are comparing things and picking the best one?

isn't the purpose of talking about opportunity cost to address idiots who neglect to do comparisons and just look at one thing in isolation?


Anonymous at 1:16 PM on January 3, 2017 | #8156 | reply | quote

Your example math is roughly what I had in mind. The value of the next best incompatible option (with no opportunity cost) is the opportunity cost for each option. Among incompatible items, this leaves one with a positive value.

But I was never that explicit about it. And I can see that when explicit like this my way is more cumbersome. And I hadn't given any explicit thought to measuring the difference between incompatible items. I can see the problems with using opportunity cost when doing that.

I actually do think of choosing a less valuable activity as destroying value though. In the case of choosing E for example, I think of it kinda like a business using the same resources to make a $2 profit instead of an $8 profit. Sure, a $2 profit is better than nothing or a loss, but the decision to do what makes $2 is still destroying $6 of potential value. So looking at E as "-6" seems right in that context.

Perhaps I'm biased by my college econ classes in this regard. One of the lessons there was that a market rate of return on capital (given your businesses risk profile etc.) should be calculated as a cost.

Anyway, I can't think of a case where changing methods would change what I actually do though. A is the best choice either way. Do you have in mind a situation / example where using opportunity cost in the calculation actually results in making a different choice?


AnonITA at 5:14 PM on January 3, 2017 | #8157 | reply | quote

> @dying you would have a good chance to do something relevant, with a chance to help, if you focused on it (and wanted to and were interested, energetic, motivated and stuff).

I don't believe this. Maybe because my standards for interested, energetic, motivated and stuff are too low.

And I don't know how to raise them.

> one of the avenues to helping with the dying problem is to get good enough at persuasion to reach people like Aubrey de Grey who are making a lot of big mistakes. if you could get him and many others to change course -- even just a minor course adjustment -- it could significantly improve outcomes (on average -- there's still a lot of variance involved). that is one of the very important projects i've been working on and gotten very little help with.

I have worked on my persuasion skills more than anything else I have done on FI. I think that I have gotten meaningfully better at it than I used to be.

But I don't think I'm good enough to give you any kind of meaningful help. And I doubt my ability to get good enough, fast enough, to make a difference.


AnonITA at 5:29 PM on January 3, 2017 | #8158 | reply | quote

> I actually do think of choosing a less valuable activity as destroying value though.

but then virtually everything that anyone ever does is destroying value.

e.g. everything i've ever written destroyed value because i didn't write something better instead.

every building ever constructed destroyed value because it was inferior to a slightly different building they could have built for the same cost.

etc

but that can't be right because the world has been accumulating value even though basically no one ever chooses perfect options.

businesses that make a $2 profit, where $8 was profitable, help with the general accumulation of value/wealth. making a profit isn't destruction.

> Do you have in mind a situation / example where using opportunity cost in the calculation actually results in making a different choice?

i don't see any value to talking about opportunity costs at all, in general. i think the concept is a reminder for people with selective attention and other bad ways of thinking. it's a parochial term to address specific biases and common cultural bad thinking.

i absolutely do see the value of comparing your profits to what you could have made buying bonds or stock index funds, or letting someone else use the capital you used, or using it better yourself. but this is just plain old comparison of incompatible options. of course you should consider alternative ways to consume a resource that are even better than your first idea.

note: you do have to watch out for inflation. you need to make a profit in inflation-adjusted dollars. so some people think they're profitable but are actually destroying wealth.

i do think those econ classes sound destructive.


curi at 5:40 PM on January 3, 2017 | #8159 | reply | quote

> I don't believe this. Maybe because my standards for interested, energetic, motivated and stuff are too low.

do you agree it's true for some meaning of those words? if they refer to larger quantities than ur standard terminology.

> And I doubt my ability to get good enough, fast enough, to make a difference.

why?

it's not as if you've seriously tried to get good at this stuff as a primary focus of your life. you've flirted with philosophy a little and are vastly better at it than most people.

you're welcome not to try if you'd rather pursue other values. but you were the one who brought up the dying problem as super important. which is a reasonable position! so why not make a try for it? b/c actually you care about a bunch of other things besides the dying problem? fair enough if so, but not really what i thought you were saying.

btw how big a difference do you care about? you could aim for at least a 1 in 100 chance to significantly help with one important sub-problem of SENS? that sounds totally doable for you in the next 10 years by quite a variety of methods including by learning a bunch of biology (doable if you actually wanted to, not while forcing yourself with gritted teeth). i bet you could do better than that if you focused on it. but maybe you'd rather hang out with your family than contribute that much? that's your choice if so, but don't think about it like "there was nothing i could do" or anything along those lines. at least tell yourself "i thought i could only make some contributions WRT dying (and WRT philosophy, and various other stuff you don't vigorously pursue) that i judged fairly small, and i preferred to do some other stuff instead".


curi at 5:50 PM on January 3, 2017 | #8160 | reply | quote

> do you agree it's true for some meaning of those words? if they refer to larger quantities than ur standard terminology.

Tentatively yes, depending on what "good chance" means.

> you could aim for at least a 1 in 100 chance to significantly help with one important sub-problem of SENS?

This is not at all what I'd call a "good chance" after 10 years. It's not clear if you intended that either, since the quote is from a different comment than when you said "good chance".

I'd characterize a "good chance" as something along the lines of: >50% chance of halting my aging process before I die.

The latter is what I don't think I even remotely know how to achieve.

The former (1 in 100 chance to significantly help with one important sub-problem of SENS) is a difference I would care about but not enough to work hard at for a long time.

> so why not make a try for it? b/c actually you care about a bunch of other things besides the dying problem?

Ya I guess, but I don't know what most of the other things are.

One thing I seem to care more about is not spending a lot of time and effort pursuing something that ultimately ends in failure.

This could be a lot social. I think I like the image that I currently have of being a successful person. It's not an entirely conventional image. I don't do conspicuous consumption type stuff for example. But I think that's mainly just because avoiding conspicuous consumption makes me more successful in other conventional ways that I care about more. If I could be just as financially successful and do conspicuous consumption too, I bet I would.

Anyway, I think my image is close enough to conventional "success" that I think that term gets the idea across. I think one thing I seem to want is to continue to be the guy who *didn't* spend 10 years of focused effort attempting something that failed.


AnonITA at 7:08 PM on January 4, 2017 | #8163 | reply | quote

> > > And I doubt my ability to get good enough, fast enough, to make a difference.

> > you could aim for at least a 1 in 100 chance to significantly help with one important sub-problem of SENS

> This is not at all what I'd call a "good chance" after 10 years.

you said "make a difference" with no qualifier about a good chance. i was talking about that. it's something a reasonable person could argue counts as making a difference.

> I'd characterize a "good chance" as something along the lines of: >50% chance of halting my aging process before I die.

that's rather aggressive. i'd be thrilled with a plan to get a >= 10% chance of that. i'd consider that GREAT and worth massive attention and effort redirected from other stuff.

i don't know how to hit 10%, but i do know how to make a difference. and i have. my discussion with Aubrey de Grey and subsequent advice offered to Aubrey/SENS was worthwhile. It was a good try that had a meaningful chance to 1) help 2) enable opportunities for me to help a lot more. (It ended up failing, but should be judged by the expected average outcome.) but even if my project had a lot of success i still didn't expect to get a 10% shot at reaching longevity escape velocity or some other AMAZING outcome.

> One thing I seem to care more about is not spending a lot of time and effort pursuing something that ultimately ends in failure.

well, yeah, people generally are risk averse cuz they only live once. so assuming no insurance, no hedges, not being rich enough to self-insure, etc, they e.g. prefer 50k over a 2% chance of 5 million (which is double the expectation value). they are right about this.

working on very hard problems doesn't allow the luxury of low variance. no one knows how to solve very hard problems AND do it with low variance! if they knew how to succeed with low variance it wouldn't be a very hard problem.

in addition to this you could be making an error here, too. people in general are extremely bad at thinking about expectation values and not being "outcome oriented" as poker players call it. (successful poker players make a lot of money off being good at this, from people who are bad at it. they also play a fuckton of hands to reduce variance. and they do bankroll management math. say your bankroll is a $1000. what entry fee for a tournament do you play? what cashgame stakes do you play? the answers are much lower than most people's intuitions. try to guess some reasonable numbers for a large NLH tournament entry fee that'd be reasonable to play regularly, and an NLH cash game big blind size that you can play regularly with the 1k bankroll.)

> This could be a lot social. I think I like the image that I currently have of being a successful person.

you definitely get less social recognitition for failures, regardless of the expectation value, b/c ppl are dumb. e.g. i don't get much social recognition from my interactions with Aubrey/SENS even though objectively i deserve credit for doing good, productive, valuable work on such an important problem that most people are contributing nothing to (or only contributing money. being able to contribute good ideas is rarer and harder to come by. we're unfortunately not a place where there's plenty of relevant expertise for hire.)

but i wasn't actually recommending 10 years on a long shot. i was arguing with the comment about you couldn't make a difference. and bringing it up in relation to the extremely high priority some of your comments placed on the aging/death issue.


curi at 7:35 PM on January 4, 2017 | #8164 | reply | quote

"make a difference" in my way of thinking (perhaps incorrect) implicitly includes "good chance". I don't think of a small chance to make a difference as "making a difference".

> working on very hard problems doesn't allow the luxury of low variance.

This is a good insight. The sort of idea that, once said, my reaction is "of course." But before it was said, I wasn't thinking about it that way.

Part of why is probably social. The problems I solve at work have the social reputation of being "very hard" problems. Maybe not brain surgery level, but plenty harder than what most people do for a living. But it's actually pretty low variance once you have the requisite skills.

So I've been kinda comfortable with the idea that I currently solve "very hard" problems while also enjoying low variance.

And when I think of switching from my current "very hard" problems to some other type of "very hard" problems, the high variance I estimate with it always puts me off.

But maybe a better way of thinking about what I currently do is that the problems require high skilled compared to the skills most people need for their job. But they are not "very hard" problems in the way that philosophy or SENS is.

So I need to think about what you propose as a transition from working on not very hard problems to working on very hard problems, instead of a transition from working on one type of hard problems to another type of hard problems.

> people generally are risk averse cuz they only live once[...] they are right about this.

+

> people in general are extremely bad at thinking about expectation values and not being "outcome oriented" as poker players call it.

I agree people are bad at this in general and maybe I am bad at it too. But I don't currently know of a criticism for thinking of i.e. a 1 in 100 chance to significantly help with one important sub-problem of SENS as a slim chance to spend 10 years of my life on.

> they also play a fuckton of hands to reduce variance.

So I see two paths forward to improve my willingness to work on very hard problems:

(1) Increase my tolerance for variance

(2) Increase the number of hard problems I can work on - simultaneously and/or in rapid succession, to reduce the overall variance of choosing to work on very hard problems in general.

I don't currently know how to accomplish either one, but do you agree with these? Any other approaches make sense that I'm not thinking of?


AnonITA at 8:46 AM on January 5, 2017 | #8178 | reply | quote

there's degrees of difficulty. maybe most people work on 20-100 difficulty stuff and you work on 250. but preventing aging deaths with SENS is like 50k difficulty. it's so hard and important that 1% is a big deal. it's more than 100x harder and the importance factor is even higher (like maybe it's 1000x more important to only 100x harder).

250 is "very hard" on a 0-500 scale (which covers most stuff people do) but isn't "very hard" compared to SENS.

i think you have a lot more variance in work outcomes than a guy who does basic hair cuts at a low end chain, cooks McDonalds burgers, or assembles iPhones. but a lot less variance than poker or SENS.

note that lots of things affect variance besides difficulty. poker has high variance for its difficulty, due to the nature of the game. I'd say chess and go are harder than poker, but they have a lot lower variance -- good players can very consistently beat inferior players.

SENS is a high variance style of project. you can't really predict when/where you get breakthroughs in original scientific (or philosophical) research. similarly, working on AGI or quantum gravity is very high variance b/c you have to try a variety of hypotheses and expect a lot of them to be dead ends, and you also don't really know how far away a solution is.

with many very hard and high variance goals, you can figure out some sub-problems/milestones that you can reach with lower variance. however they may be things like testing a hypothesis, with the possibility that all you do is establish that's not the answer, something else will have to be tried. sometimes there are sub-projects with much more fulfilling worst-case outcomes.

the broad category of improving your skills works well for low variance sub-projects. you could become a better writer as a milestone towards working on a hard philosophy problem, and the variance on gaining some writing skills (that are like solidly good but not way above a lot of other writers) is reasonably low variance.

getting to 80% of the skill level at something, which 100,000 living people already have, is usually reasonably low variance to achieve.

knowledge work where creativity is required is generally way higher variance than labor. variance is a big concern for many software development projects, but not a big concern for furniture movers.

some less ambitious software development projects are done pretty reliably, e.g. coding Mario Run was well within Nintendo's abilities on the coding end (it was more likely to have design problems and be cancelled because it wasn't fun than because they had trouble getting the 2d platforming code to work). but even seemingly fairly mundane software development can fail badly, e.g. the obamacare website debacle.

> I agree people are bad at this in general and maybe I am bad at it too. But I don't currently know of a criticism for thinking of i.e. a 1 in 100 chance to significantly help with one important sub-problem of SENS as a slim chance to spend 10 years of my life on.

i wasn't recommending that! i was contradicting the can't make a difference comment.

it only takes 100 people spending 10 years on such chances to get good odds to significantly help with SENS. that's a big fucking deal. 1% of that is still a lot, a worthy contribution. and that's true even if the other 99 people don't exist so you have much higher variance. your contribution is the same whether the other 99 people work on it too or not. i'm guessing if 100 people worked on 100 leads, and you were one of them, and 5 of the leads bore major fruit, and you did one of the fruitless leads ... you'd still feel proud and accomplished as part of the overall project. but you'd have personally done the same work as if the other people didn't exist, trying one worthy lead that ended up not panning out.

there are practical problems with that kind following a failed lead. it can be hard to get funding, especially if you do it alone instead of as part of a bigger overall effort. but setting that aside, it is valuable work.

but still i do think there are better options. the world needs philosophers more than anything. and variance on a project to learn Popper and Rand is an interesting topic in general. a lot of people fail. there are doubts about how much they really cared and tried. but there's also concerns they are failing due to common irrationalities that make it hard. anyway i think your personal odds of successfully learning Popper and Rand to a high, valuable standard are a hell of a lot better than 1% if it was your #1 priority that you focused on and tried for. also the outcome of this project isn't independent of the outcomes of lots of other projects. the main reason you would fail is because of failure to overcome some thinking flaws that would also have prevented successful completion of most other major, hard, important projects.

being a great philosopher makes some things low variance. for example, i can go to whatever field or project i want and find and explain a lot of mistakes with high reliability (especially if people will answer questions, explain what they are doing, share information). and i can create action plans to fix them and do substantially better. however there are some problems which make this skill hard to utilize: 1) people who don't want my help and won't listen to me 2) people who sabotage perfectly good, actionable solutions because of their irrationality.

the problems of attention and irrationality are hard.

i'm not talking about solutions that people don't understand, that are too advanced for them. i'm talking about simple solutions that people could do if they took my word for it being good and were willing to do it. (i can give simple explanations for why it's good but understanding the full arguments and answers to all the rival philosophy claims is beyond most people's understanding). i think a good example here is hospital checklists. there's no question that people could use checklists. it isn't too hard or complicated for them. but there's a lot of resistance to using checklists by many doctors. it's a legitimately actionable plan that doesn't depend on them learning some hard new skill and maybe failing to learn it. checklists aren't that fucking hard. people just sabotage it. anyway what i'm saying is i can come up with "checklists for hospitals" type solutions for pretty much whatever field i care to look at, with *low* variance. philosophy enables that much. but then it runs into the problems of people not listening, refusing to do it, etc, which prevents getting all the way to reliable huge wins in the world.


curi at 2:38 PM on January 5, 2017 | #8179 | reply | quote

> i'm guessing if 100 people worked on 100 leads, and you were one of them, and 5 of the leads bore major fruit, and you did one of the fruitless leads ... you'd still feel proud and accomplished as part of the overall project.

Interesting hypothetical. I think there are two interesting variations with different answers! The two variations I'm thinking of are:

- The 100 people working on 100 leads are organized, I'm a recognized part of the organization, and which leads to pursue and who pursues them are determined in some reasonable fashion by the organization as a whole. In that case, I think your guess is correct - I would feel proud and accomplished even if the lead I personally worked on was fruitless.

- The 100 people are not organized. It's just 100 people individually pursuing whatever lead they think is best to pursue. So I just individually pick a lead and pursue it. In that case, I think your guess is incorrect - I wouldn't feel proud and accomplished if the lead that I personally worked on was fruitless. I'd be elated about not dying because someone else's lead panned out, but ( wouldn't feel in any way responsible for it or successful because of it.

I think the reason for feeling differently is that in the first case, I made a choice to join a recognizable project that pursued 100 leads - which was successful as a whole. The thing I participated in - the organization - was a success. In the second case, I only made a choice to pursue 1 particular lead, which was a failure.

> i think your personal odds of successfully learning Popper and Rand to a high, valuable standard are a hell of a lot better than 1% if it was your #1 priority that you focused on and tried for.

I have no reason to dispute this, but I don't currently care much about learning Pppper and Rand to a high, valuable standard as such. Definitely not enough to make it my #1 priority to focus on and try for. I only care about it as a means to i.e. not dying. So the part I care about is still very hard / high variance.

That leads me back to the question of if I should work on one or both of:

(1) Increase my tolerance for variance

(2) Increase the number of hard problems I can work on - simultaneously and/or in rapid succession, to reduce the overall variance of choosing to work on very hard problems in general.


AnonITA at 6:07 PM on January 5, 2017 | #8181 | reply | quote

yeah i meant it's organized as one big project, on purpose. not just 100 smaller independent projects happen to accomplish this.

but basically you're wrong. if reasonable ppl including advocates of Y think that X and Y are two roughly equally good and valuable leads, and your pursue X and someone else solves the problem with Y, you did something about as important as him. it had, as best either of you knew, about the same chance to solve the problem. (this is an idealized scenario. in practice the ppl working on Y often think their lead is the best one, whether due to bias or having some insight. and some other stuff can matter. but still the practical result is similar: working on a valuable lead is valuable even if it doesn't pan out, just getting all the money in the middle preflop with the better hand hand is a good play even if you then lose.)

work on lead X is valuable whether or not there is e.g. a large group funding work on Y at the same time. the value is similar either way. you do want to be careful that e.g. your work doesn't get duplicated pointlessly by someone else. there are ways to approach that in either case and they are about equally effective (it's easier to get the other 99 ppl in this one organization not to duplicate your work due to ignorance you'd done it but that's still a small fraction of the ppl in the world. and if it's a tiny field with not many ppl doing stuff, it's not that hard to communicate to the others. just like the group of 100 would check what some independent guys had also done and maybe skip investigating a lead that they judged was already tried well enough).

> (1) Increase my tolerance for variance

you shouldn't *tolerate* variance. you should understand when and why it's bad. and when it's ok, and specifically what problems it does and doesn't cause and how to mitigate the problems. and *think rationally about it*, which people mostly don't do (top poker players being a standout group that's actually good at it).

the way best way to get good at this includes learning Popper and Rand. (the poker players generally have too fragile and narrow a skill and don't know how to apply it to thinking about anything.)

> (2) Increase the number of hard problems I can work on - simultaneously and/or in rapid succession, to reduce the overall variance of choosing to work on very hard problems in general.

the way to do this includes learning Popper and Rand.

---

anyway like i was talking about there are lower variance ways to make progress in hard problems. such as learning Popper and Rand. that's a lower variance way to approach the problem of dying and many other hard problems.

note that variance and low odds are different. if you can guarantee you increase the odds of not dying from .1% to .2% that'd be zero variance about how much progress you make, how much you accomplish. as opposed to testing a hypothesis which probably won't pan out which has high variance.

not dying remains a hard problem but there are ways to approach it where you're making reasonably steady progress and getting useful stuff done, that aren't all or nothing. and they have benefits for the rest of life too, so it's just a good idea cuz it makes all your life better while also being the best way to approach not dying. you can do things like increase your ability to solve problems and think well, and apply this to issues like SENS or anything else, and the variance on this isn't that bad. but even if you're sure to be productive and get somewhere (instead of pursuing some leads that hit dead ends) the odds of not dying are still low.


curi at 6:38 PM on January 5, 2017 | #8182 | reply | quote

> but basically you're wrong.

I agree - intellectually. But the issue was about how I would feel. It was:

> i'm guessing if 100 people worked on 100 leads, and you were one of them, and 5 of the leads bore major fruit, and you did one of the fruitless leads ... you'd still feel proud and accomplished as part of the overall project.

What I'm saying is, if I worked independently on a lead of my own and it didn't pan out I'd *feel* like a failure. Even though intellectually I have no crits of your arg that I should feel proud and accomplished.

And I think a good portion (though perhaps not all) of the reason I'd feel like a failure despite intellectual arguments to the contrary is social. I think that other people would see the result as a failure rather than something worthy of being proud and accomplished. And they'd be unlikely to hear or care about the intellectual arguments for why I should feel proud and accomplished.

And I think the reason that matters to me is that I like the image of being a successful person, and I don't want to give that image up.

> note that variance and low odds are different.

gp. I think the problem I've been calling variance is about low odds. I don't think I'm worried about ex: the variance of working on a lead that visibly produces say 5 years of extra life instead of a hoped for 500 years of extra life. I would still feel successful if everyone can see that I worked on a project that "only" extended human life by 5 years. That's still a super huge success! What I'm worried about is the very large probability of working on a lead that completely fails and gives no extra life.

Worse, I think I'm worried even about working on a lead that could with reasonably high odds give me personally 5 years of extra life *but not visibly*! Say for example that by working hard on a SENS lead that doesn't pan out, I learn enough about the healthcare system & health in general to make better choices than I would have otherwise made wrt hospitals, doctors, treatment options, etc. and because of that I live 5 more years than I would have lived. Pretty plausible scenario actually. I'd be glad to have those 5 extra years, but it would still *feel* like a failed effort because nobody else would see that what I was working on led to those 5 extra years of life.


AnonITA at 11:48 AM on January 6, 2017 | #8183 | reply | quote

so rather than figuring out how to work on the problem of dying, work on the problem of learning on how to live? iow, learn how not to be a secondhander. that's a pain point you could want to fix.


Kate at 4:31 PM on January 6, 2017 | #8184 | reply | quote

> that's a pain point you could want to fix.

Yes, I could want to fix it.

In some sense just spelling it out plainly and honestly is helpful. It's one thing to vaguely know I've got secondhanded ideas. It's another to clearly identify one and bring it into the light of day.

But now the question is, do I actually want to fix it enough to do something effective to that end?

I don't know yet.


AnonITA at 6:42 PM on January 6, 2017 | #8185 | reply | quote

I've been thinking some more about this.

Why don't I know if I want to fix this particular secondhanded idea about wanting to be seen as successful?

Liking and wanting the image of a successful person plays a role in the best explanation for some things I've done in life that I consider objectively good.

It's true that there's probably other motivations that could have worked for those things.

Also, there's probably even better stuff I could've done in life than what I did from the motivation of wanting the image of a successful person.

And finally, just because that motivation played a role in good things I did in the past doesn't mean it has to play a role in the future.

BUT...

How much I'm currently motivated by things other than to be seen as a successful person is not clear to me. I can't put my finger on any right now. I kinda suspect there's very little.

Not dying is the most powerful one I have thought of so far. And yet, as this discussion has revealed, that motivation withers in the face of impairment to the successful image motivation.

So I think it's possible that if I stopped caring about being seen as a successful person, I'd stop doing much in life altogether. That's a scenario I judge to be worse than doing a bunch of things that lead to being seen as a successful person.

In other words, if I could somehow wave a magic wand and get rid of this second handed idea of wanting to be seen as a successful person (but make no other changes), I rather suspect doing so would make my life worse.

So at a minimum, I think it's not wise to just start trying to learn not to be a secondhander. Maybe that needs to come later.

First I need to figure out what motivates me without contributing to an image of success. Or how to cultivate such motivations in their absence. Or something like that.

Only then do I think it will be safe to try to get rid of the successful image motivation.

Make sense?


AnonITA at 8:58 AM on January 7, 2017 | #8187 | reply | quote

> And I think a good portion (though perhaps not all) of the reason I'd feel like a failure despite intellectual arguments to the contrary is social. I think that other people would see the result as a failure rather than something worthy of being proud and accomplished. And they'd be unlikely to hear or care about the intellectual arguments for why I should feel proud and accomplished.

*whose* opinions do you *respect*, who thinks this way? or are you non-specifically trying to impress no one in particular that you don't respect the judgement of? (and don't even think about whether you respect their judgement.)

> Worse, I think I'm worried even about working on a lead that could with reasonably high odds give me personally 5 years of extra life *but not visibly*!

bad ideas cause big problems in one's life... good example. *this is worth changing*.


curi at 3:00 PM on January 7, 2017 | #8188 | reply | quote

> How much I'm currently motivated by things other than to be seen as a successful person is not clear to me. I can't put my finger on any right now. I kinda suspect there's very little.

idk about "very little". then why would you participate at FI at all?

> So at a minimum, I think it's not wise to just start trying to learn not to be a secondhander. Maybe that needs to come later.

learning how to live firsthanded is part of learning not to be a secondhander. learning replacement ideas is a standard part of learning about mistakes. e.g. creationists need to learn not just that they are mistaken but also some new concept of the world to use.

finding out mistakes should lead people to take an interest not just in rejecting the mistake but also finding out a better idea.

> Make sense?

i think you can work on stuff in parallel and there will be connections. e.g. some idea about what's wrong with being a second hander will help give you an idea of where to look for firsthanded motivation (cuz you see what's going wrong there better, it gives you a clue about what to do instead).


curi at 3:07 PM on January 7, 2017 | #8189 | reply | quote

> *whose* opinions do you *respect*, who thinks this way? or are you non-specifically trying to impress no one in particular that you don't respect the judgement of?

I think it's fairly specific. There are 3 groups of people I think I care about seeing me as a successful person:

- Family (both immediate and some extended)

- Co-workers and managers at my job

- Fellow members of a specific business association

I think I'm currently viewed as a successful person by all three groups.

> idk about "very little". then why would you participate at FI at all?

Earlier in this thread you stated "you've flirted with philosophy a little and are vastly better at it than most people." Two key things about that:

- I haven't ever got beyond the hobby level of effort with FI. I haven't devoted the kind of focus and energy to it that I have devoted to my job or a few other things in life. And when I contemplate making a higher level of effort at FI, or have the opportunity to do so for a time like when I'm home but off work for many days in a row, I lack the motivation to do it.

- I think being more intellectual than normal is a component of the particular image that I want to have with the groups that I care about. For example, I can say things that people in those groups will find surprising and "profound" in a good way - stuff they don't hear from anyone else. I do it often enough that they notice, remember it about me, and will sometimes comment positively about it. At the very least, FI gives a good return on that kind of skill per unit of effort compared to other "intellectual" things that I could do or did before FI.

But I also think I'm at the point of diminishing returns. If I increase my rate of "profound" statements much I think my image would go from "the successful guy who says some of the most interesting things," to "that egghead who lets intellectual naval gazing ruin his true potential." It's a fine line; I think I've recognized it at some points in the past and backed off saying things I'd otherwise say.

To be clear, I'm not saying that I think image is my only motivation to participate in FI. Or that it's my primary motivation. It could be 5% or 95%. At this point I don't know how much it is.

But however much it is, I don't want to disturb it until I have something effective to replace it with. Lots of people quit FI. I think that could be a real danger for me too.

Lots of people also quit trying to be successful (or never start) and just sorta coast through life in general. If they're young or poor it's commonly called slacking. If they're old and at least somewhat rich It's commonly called retirement. Whatever you call it, I think it's an important danger for me to avoid.


AnonITA at 5:59 PM on January 7, 2017 | #8190 | reply | quote

> There are 3 groups of people I think I care about seeing me as a successful person

those all sound like groups that routinely have *false* opinions.

there's something i've never understood about putting popularity before truth. i think it's what Rand calls social metaphysics. judging ideas by people's opinions of them and the status of those people, instead of whether they're true or false.

do you have a system of rationalizations to feel OK about this non-truth-oriented approach? or you hadn't thought about it a lot before? but now that you've looked at it more, how can that be OK with you? just b/c you're worried about dangers of changing? the dangers would be a reason to figure out good ways to change that satisfy your (reasonable) concerns, not a reason to just do nothing. (some concerns you expressed about changing were reasonable issues. you may have some unreasonable concerns about changing too.)

i think it's partly an issue of taking the world as you find it. that's what most people do. if it turns out a wife wants X, Y, Z and then you can have a wife and get a bunch of stuff you want (activities, conversations, lifestyle, etc) then people do that. they just look at it like "ok do these things, get what i want".

i judge the price and lose respect for the asker. i don't just look at the deal and think "ok i want a wife really bad. i'll do what it takes" like ppl normally think. it's not just a direct cost/benefit thing to me. i look at it and go "if she's such a bad person to disrespect the truth and be really socially oriented and conformist, then the interactions with her aren't going to live up to my expectations anyway, so it can't be a good deal". if someone wants something awful as their price, i start questioning the value of what they're offering.

i agree about diminishing returns on profound statements. very sad... that's another instance where i'd look at the demands of the ppl i was dealing with, see the demands are very bad (limit good ideas you bring up), and then start doubting the value they offer in return for that awful price.


curi at 7:03 PM on January 7, 2017 | #8191 | reply | quote

> do you have a system of rationalizations to feel OK about this non-truth-oriented approach? or you hadn't thought about it a lot before?

I hadn't thought about it a lot before; especially hadn't thought about it applying to me.

Thinking explicitly about what made the difference between feeling good or bad about working on a SENS lead revealed a lot about me I wasn't previously aware of.

Up to that point I didn't think I cared much about other people's opinions. Explicitly I did (and still do) espouse a mostly "don't give a fuck" attitude about the opinions of others.

I thought of what I did in life more along the lines of: Sometimes I defer a little to social conventions. But it's objectively a pretty minor aspect of how I live my life. Mostly, I do what I think is right. I could be mistaken, but the opinions of others aren't what's usually driving either the mistakes or the good stuff I do.

So I think one system of rationalizations was about hiding from myself the fact that the opinions of others actually mattered so much to me.

I don't know if I have other rationalizations in place about being OK with the non-truth-oriented implications of caring. I'm cautiously optimistic that maybe not so much. Because I was pretty good at hiding the fact I cared a bunch, maybe I didn't need to build up a ton of rationalizations about it being OK to care. But really...I don't know yet cuz I haven't been looking long.

> the dangers would be a reason to figure out good ways to change that satisfy your (reasonable) concerns, not a reason to just do nothing.

I agree. I don't want to just do nothing.

I need to develop the motivation to do something. Motivation that doesn't rely on being a success according to others.

And I need to figure out the right thing(s) to do with that motivation.

> if someone wants something awful as their price, i start questioning the value of what they're offering.

Makes sense.

One problem is: because I was good at hiding from myself that I cared, I don't know how much is actually being demanded as a price and how much is just my assumptions about it.

It's possible that it's all being demanded and I've just been good at figuring out people's price and meeting it while hiding that from my explicit consciousness.

It's also possible that lots of it isn't being demanded at all, never was, and I'm just conforming to a bunch of unquestioned assumptions about what people demand.

I have preliminary indications in both directions. And that its different with different individuals and groups.

Even this is dangerous though. Suppose I figure out that lots of people I was acting as if demanded an image of success from me, actually don't demand that. But that (assumed) demand was my motivation for making a lot of the effort in life that I made, and now it's gone cuz at worst now I only care about the real demands. Does my motivation similarly evaporate and I start living the life of a slug? Maybe!


AnonITA at 4:21 AM on January 8, 2017 | #8192 | reply | quote

boldly, forward

the **big picture** is this:

thinking is always dangerous.

new ideas bring new problems.

people have to choose: a static life or the beginning of infinity.

mostly today in the west they try to compromise: they think within limits and pretend it's not a static life. like: they only think within a box, and they call a certain part of the box "outside the box thinking".

the only way to deal with a succession of new problems is to go full speed ahead with learning and wealth creation to increase your problem solving power ASAP to cope with whatever goes wrong. it's that BoI attitude or else you can stand still and pray. or the third option is some kind of unprincipled, mixed approach combined with rationalizations to lie about what you're doing.

there's only one option that makes any sense. arbitrary, irrational limits cannot help. they can limit progress but they don't actually prevent problems. and standing still means waiting to die -- aka retirement.

life doesn't come with guarantees of success. so what. try anyway.

**nothing can get rid of problems**. stand still or limit your movement and problems will come up anyway. and closing your eyes like your static memes are adapted to cause doesn't help either -- ignorance isn't bliss. *so the only thing to do is do your best to solve problems, to learn, to make progress.*


curi at 4:53 AM on January 8, 2017 | #8193 | reply | quote

I think we agree on much of the **big picture**, but one thing stands out to me as a possible misunderstanding or disagreement.

The metaphor of crossing a desert mentioned in #8170 seems appropriate in some ways to the way I think. I know some of the dangers involved with desert hiking like rattlesnakes, jumping cactus, scorpions, cliffs, javelina, coyotes of both the human and animal variety, heat, dryness, UV, fatigue, sandstorms, lightning and, if it does rain, flash floods.

You gotta both anticipate and be on constant lookout for those dangers. But if you spot a danger it doesn't mean you should stop short of it, pitch a tent and attempt to live the rest of your life wherever you stopped. When crossing a desert, permanently stopping is pretty near certain death.

> the only way to deal with a succession of new problems is to go full speed ahead with learning and wealth creation to increase your problem solving power ASAP to cope with whatever goes wrong.

Maybe we disagree about this "go full speed ahead" part. It's definitely not the phrase I would use. The closest thing to it I'd agree with is "go at the fastest prudent speed for the dangers you face".

Sticking with crossing a desert metaphor, when you spot a danger you do have to figure out how to deal with it appropriately. You shouldn't apply the same strategy to all dangers. If your policy is to just charge full speed ahead that's also likely to get you hurt or killed. Sometimes the best answer is to find some cover, hunker down and wait a finite period of time until the danger passes on its own. Sometimes you should press on, but slow the pace. Sometimes you should take a detour from your original path. Sometimes you gotta run for higher ground. And sometimes, it's best to deploy a gun on the problem.

Maybe the desert crossing metaphor is not as apt to the big picture as I think it is. One thing is a desert hike usually has a known, planned end point whereas growth of knowledge doesn't.

But anyway, can you clarify your position on how "go full speed ahead" applies to dealing with the variety of dangers you may recognize?


AnonITA at 10:17 AM on January 8, 2017 | #8196 | reply | quote

Just realized the comment reference above didn't work. Full link to the original desert crossing metaphor is:

http://curi.us/1896-fallible-ideas-unstructured-discussion#c8170


AnonITA at 10:19 AM on January 8, 2017 | #8197 | reply | quote

gradualism

> But anyway, can you clarify your position on how "go full speed ahead" applies to dealing with the variety of dangers you may recognize?

I'm guessing that "full speed ahead" here means "gradualism".

In FI 13289, Elliot wrote (bold mine):

>> so there is this logic of epistemology (which works by evolution) where you need like 99% preservation and 1% change in order to best make progress. so views which see it this way are called gradualist, because it's nothing like 50/50 preservation/change or 1/99 preservation/change, it's mostly preservation. but this maximizes progress! it's not going slow, **it's going as fast as possible.**

>> trying to rush and making errors at a higher rate doesn't actually work. it doesn't get you where you're going faster. there's no shortcuts. in some ways, the name "gradualism" is misleading since **it's the fastest method of knowledge creation.**


Alisa at 2:16 PM on January 8, 2017 | #8199 | reply | quote

life is a RACE against problems

progress is a RACE. problems will DESTROY you, civilization, the Earth. the deadlines are largely unknown (e.g. when a meteor will hit the Earth unless we have a lot more knowledge and wealth to deal with it), but our ignorance doesn't prevent the deadlines from existing.

on an individual level, besides death, injury and illness, people face risks like a bitter divorce, being fired, or your children hating you and having awful, failed lives dominated bad thinking.

how can any married person address the risk of divorce? basically one can either hope to get lucky or one can improve and apply his capacity to solve problems. how much progress makes one safe from a nasty divorce? there is no safety in life. but it's the more the better. there is an endless succession of problems coming your way, on a schedule you don't know, and they each have a level of difficulty in many different aspects, and if you don't make enough progress to have the skill/resources/etc to deal with it whenever it comes along then you will fail and suffer. it's a RACE. if you're too slow you get fucked.

AnonITA expressed doubts about his ability to make progress. that is no reason to hesitate. do your best. try. or lose the race by default. (or think of it as many races, and lose them all by default.)

there is no reason not to try. there is no safety in hesitation. there is no safety in waiting. there is no safety in standing still or not trying. of course this is compatible with sleeping, taking breaks, sheltering in a storm, etc, all of which help maximize your problem solving capabilities (e.g. by protecting resources from destruction in a storm, leaving you with more power to get problems solved next week then you'd have if you didn't take shelter). shelter in a storm solves a problem and is totally different than an attitude of fearing or hesitating to make progress.

shelter in a storm isn't fear of the *unknown*, pessimism about making progress, doubts about your own abilities, doubts about your own mind, or pessimism about problem solving. shelter in a storm is a solution to a specific, *known* problem.

race car drivers turn to avoid crashing into the wall rather than just holding down the gas for the entire race to naively try to go faster. runners save some energy for the last lap of their race in order to get a *faster* time, rather than naively trying to sprint the entire e.g. mile.


curi at 3:01 PM on January 8, 2017 | #8200 | reply | quote

@#8197 you can't short link to other threads. i don't think i should add that feature cuz i think ppl seeing a short comment link would expect it refers to a prior comment from this discussion, so it'd be misleading (even though i'm using global comment ids instead of starting counting from 1 for each thread like some other sites do).


curi at 1:35 AM on January 9, 2017 | #8203 | reply | quote

> AnonITA expressed doubts about his ability to make progress.

That wasn't my intent. Can you quote what I said that you think indicated such doubts?

I'm actually confident about my ability to make progress. As I said in #8150:

> I think there is no problem I could not solve if I live long enough.

Further, I'm not just confident in my *ability* to make progress. I think I *have* been making substantial, measurable progress in terms of what curi mentioned, "skill/resources/etc". I think my progress has been consistent for very many years and is still ongoing. I don't think it's been a fluke, luck, or some other kind of anomaly.

One set of problems / doubts I think I tried to express and actually do have are around winning the RACE vs. avoiding a crash-and-burn. Or ambition vs. over reaching. Or "if you're too slow you get fucked" vs. "you need like 99% preservation and 1% change in order to best make progress". I'm not confident about my judgement of those questions at the margin. Some cases are super clear to me, but the ones I've been talking about in this thread aren't.

Another set of problems / doubts I have are around what areas it's best for me to work in, and making tradeoffs about that. Lots of the progress I've made to date has been in areas that aren't directly about philosophy, or aren't even intellectual. That doesn't make such progress unreal. Should I continue that work cuz it's going so well? Should I try to just maintain what I have there and shift some focus? Should I risk losing some progress in those areas in order to make more progress elsewhere?

Those are the actual doubts that I have. Not about my ability to make progress.


AnonITA at 9:12 AM on January 9, 2017 | #8204 | reply | quote

doubts

AnonITA wrote:

> Lots of people quit FI. I think that could be a real danger for me too.

i interpreted some substantial doubts here!

curi wrote:

>> i think your personal odds of successfully learning Popper and Rand to a high, valuable standard are a hell of a lot better than 1% if

my meaning here was a lot more than 1% but also still a lot less than 100%. you replied agreeing so i thought we were on the same page with having some doubts about this.

one common way a learn Popper/Rand project fails is people start it but then quit. so the prior doubt about maybe quitting FI is relevant here.

AnonITA wrote:

> Further, I'm not just confident in my *ability* to make progress. I think I *have* been making substantial, measurable progress in terms of what curi mentioned, "skill/resources/etc". I think my progress has been consistent for very many years and is still ongoing. I don't think it's been a fluke, luck, or some other kind of anomaly.

your history of progress does NOT indicate you would have success making UNBOUNDED progress. lots of people make BOUNDED progress (progress within certain limits, e.g. socially normal types of progress like career and wealth progress or learning to cook). that's common and different.

unless there are some specific details i don't know about that indicate otherwise, i think your history of progress is broadly compatible with inability to make unbounded progress. and there are many other people with a similar history of various progress in their life who are like super irrational and kinda hopeless if you start talking philosophy or some other kind of progress outside the box they live and think in.

AnonITA wrote:

> That doesn't make such progress unreal.

it makes it not a great indicator of your ability to make ALL types of progress.


curi at 3:24 PM on January 9, 2017 | #8206 | reply | quote

@quitting FI, I have been thinking in terms of giving up and quitting on progress being different than not being able to make progress. Like for example, I think DD could make progress if he wanted to, he has just currently quit/given up on FI. He didn't lose his ABILITY to make progress, just his DESIRE to do so at least as regards FI. Or another example, I quit going to lefty atheist groups, but I am still able to go to such groups if I wanted to.

@Popper / Rand, again I thought that quitting trying to learn them is different from not being able to make progress at it. I do think the chance I'd make progress with learning Popper and Rand is 100% if I had a desire to do so and infinite time (solve the death problem).

You seem to be treating a decision not to do something as the same as a lack of ability to do that thing. Have I got that right? If so, why do you treat them the same?

> your history of progress does NOT indicate you would have success making UNBOUNDED progress.

What do you think would stop me, other than death?


AnonITA at 6:36 PM on January 9, 2017 | #8207 | reply | quote

long story short, i think people who are able to make progress do.

DD is stuck because he can't deal with some problems. it's not that he doesn't want to. *he doesn't know how*.

people quit trying to learn Popper/Rand/FI/etc b/c they don't know how, they're stuck, what would normally be called "they can't do it, they failed" (even though given a million years yes there are ways to get unstuck that are theoretically possible)

another example: i could learn to play a musical instrument. i don't want to. that's totally different than people who want to do something like understand capitalism but fail and fail and fail until they give up because of their flaws NOT because they realized capitalism isn't so important after all and they have others uff to do.

he's capable in the sense that given the right knowledge he would succeed. his brain hardware is capable. but he doesn't know how, he's stuck, and at this point it looks grim because he's gone downhill a lot.

what happens to people who get stuck too long is it's psychologically difficult. and they often stay optimistic for a while and keep trying, but eventually start rationalizing their failure, making excuses, coming up with a pretty narrative they like better than their failure, shifting blame, etc. and once they do a large amount of that they are really fucked. it's still fixable given a million years but it's so far away from making significant progress.

this is totally different than you quitting those atheist groups. you didn't quit those because of a failure. i agree you could go back. you weren't stuck. DD didn't quit FI in that way. he got stuck and started becoming irrational about it after years of not making enough progress (he actually held out an exceptionally long time after he was in trouble).

> What do you think would stop me, other than death?

giving up or getting stuck. people get stuck. for people who tried at some point, getting stuck is the main reason they give up.

if you're trying to say that giving thousands of years to read every popper book hundreds of times you'd eventually be able to figure it out, i agree. IF you had the patience, which maybe you would if you had that lifespan. but like being realistic, in the world the way it works now, stuff happens to people like: they read several popper books 1-2 times, don't get it, try to discuss several times over the period of a few years, get stuck, don't figure it out, give up. from my perspective that's totally possible to happen with you. i don't know if you're thinking about what you could do with like thousands of years of study (giving plenty of time to e.g. take 10 year breaks to watch TV until you get bored and go back to trying philosophy again), or what's more realistic to happen. there's an important difference here about like whether you could learn Popper rather well in the next 2 years or not if you tried to. like if it'd take 2 years, 20 years, or 200 years, those are pretty different sorts of capability of doing it.

---

some summary:

1) you're wrong about DD and most other people who quit FI. they didn't quit it like you left those atheist groups, just choosing to do something else. they failed and basically only quit b/c they got tired of failing continuously over time.

2) there's a huge difference btwn being able to learn something with X resources and with 100000X resources. it's bad to blur this distinction. it makes you sound a lot more capable than you really are. you should be putting more attention towards how much resources (like time) it takes for you to do something, cuz that does matter, cuz resources are scarce in real life. requiring too much resources to learn Popper actually makes the difference between you doing it or not doing it in your life. (one of the things you could consider doing is a test run where you seriously try to learn Popper, TCS or something else for say 2 months. and then you could see the rate of progress and how much resources it costs and get a better understanding of how viable an option philosophy is for you. like you don't actually really know how well you can do it, how capable you are.)


curi at 11:35 PM on January 9, 2017 | #8208 | reply | quote

OK, I think I'm in broad agreement about the current state of reality. Regardless of what I might be able to do with infinite time, I don't have infinite time. So the important question in reality is: how much progress can I make in the time I expect to have?

I agree my progress in that context is bounded. Is it not bounded for you as well though? I get the sense you think you and I are different on this issue of making unbounded progress, but I don't understand what you think the difference is.

> one of the things you could consider doing is a test run where you seriously try to learn Popper, TCS or something else for say 2 months.

I think it'd take major, disruptive reorganization of my life to do that. Which as I said prior in this thread is not out of the question. But when I had the opportunity to do that sort of thing for 2 weeks instead of 2 months without major disruption, I didn't want to.

One theory I have is that 2 weeks just wasn't long enough. I always have a backlog of "minor" things I want to do. Projects to fix minor things, organize, optimize, tinker, TV shows I want to watch, etc. It kinda irritates me that these things don't get done, they pile up and then when I have work breaks I tend to work on them.

If my break had been 2 months or 2 years instead of just 2 weeks, I might have got through those to the point that I'd want to do some serious philosophy study. I was more interested in doing philosophy stuff toward the end of the break than the beginning, but it was too soon time to go back to my regular work.

But maybe that theory's false. Maybe I would just find an endless list of minor or major diversions and distractions, never getting around to serious study of philosophy. One major indicator in that direction was discovering how big a role a successful image plays in my motivation. I could easily want to focus on something which would appear more successful instead of philosophy.

Short of concocting some kind of major lie I don't think I can just put my current life on hold for 2 months and try something different. I'd like to try that if I could, but I don't think I am in a position that allows for it.

I think another key consideration is I expect things to change on their own in the next few years for a variety of reasons. I'd be super surprised to have this same dilemma in, say, 10 years.

So the situation doesn't seem to me to be stuck forever vs. change. It's wait for change to happen on its own, a clearer path to become apparent, or attempt a possibly damaging reorganization now.


AnonITA at 8:00 AM on January 11, 2017 | #8209 | reply | quote

most people get stuck trying to make some bit of finite progress (e.g. understand Popper). this indicates their inability to make unbounded progress! (without way more resources).

lots of people don't want to. that's also a problem they have that gets in the way of unbounded progress. (and is fixable with tons of resources). note this not-wanting-to is often an excuse to hide inability.

for example, i think kate sams and rami rustom are stuck and failing regarding learning Popper. whereas Justin and Alan are capable enough regarding Popper, not stuck. and you I think are a maybe, and unknown. i don't know which way it would go if you prioritized it more. i also don't know how honest and open to reconsideration your prioritization is or if there's really some other issue that it's serving to hide.

> I think another key consideration is I expect things to change on their own in the next few years for a variety of reasons.

you sell yourself short. they won't change on their own. you mean you would do lots of stuff successfully! you're writing your own actions out of the story because it seems natural and expected to you that you'll do them successfully, so in your mind they've become a default. life isn't automatic and there are lots of ways you could fuck up the next few years, you just (reasonably) don't expect to. imagine some idiot had your life. try to think of all the ways he could fuck it up and not get the sort of situation 10 years from now that you expect to get. there's a million ways to mess it up. it won't happen automatically on its own.

just because some of the projects you expect to wrap up have some time limits doesn't mean their resolution is automatic. if you fucked them up enough they'd have lasting consequences after what you currently expect to be the end date! if you cause some disaster there's no reason it couldn't lead to a succession of new disasters so that you're super busy and overwhelmed 10 years from now.

i'm not expecting bad stuff like that to happen. but the fact that it COULD happen demonstrates that you're actually doing stuff to get the outcomes you have in mind, they won't just happen because merely time passes.


curi at 3:00 PM on January 11, 2017 | #8210 | reply | quote

> life isn't automatic and there are lots of ways you could fuck up the next few years

True, but I don't think I conveyed the nature of my situation correctly. There's lots of aspects but the easiest to describe is financial.

Over the next 10 years my financial situation could change for the better or for the worse or somewhere in between / mixture. Depending on both my actions and circumstances beyond my control. But in most of the foreseeable circumstances I'd be far better positioned to i.e. take 2 months to seriously focus on learning Popper or Rand without causing additional problems.

One way to think of my current situation is my work is highly time consuming, highly profitable, fairly fragile but because of how I set up my life it doesn't involve a lot of downside risk at this point assuming I don't fuck other things up like some idiot.

If the job "breaks", I would have plenty of time & money for a 2 month or 2 year try at philosophy followed by a reasonable try at something new if philosophy doesn't work out. I have lots of savings, some other income sources, don't spend near what I currently earn, and have reasonably high-value skills. My wealth might go down for a while, but I wouldn't be fucked.

If the job doesn't "break", pretty soon I will have accumulated so much wealth I could quit & not worry about if I earn money from a job or job-like contractor type work ever again. By some reasonable formulas I am there already. But I think I currently have too many unknowns and also that it's best to use more conservative formulas for determining this. But the unknowns will resolve themselves and as my wealth continues to rise I'll surpass the thresholds in the conservative formulas too.

Sure it's possible I could do dumb stuff that would leave me both time and asset poor again (that's how I started out my career). I'm not planning on that; I'm careful, but it's possible.

Or something else could happen in my life that's not directly finance-related but would nevertheless change things. I don't judge any of the likely scenarios would leave me BOTH time and asset poor, and if they left me time poor it wouldn't put me off the path to be both time and asset rich in a few years.

What I don't currently think is right is deliberately doing something I have good reason to think will interrupt my wealth accumulation cycle. If it breaks on its own, fine. Or if it doesn't break on its own it gets to a point where I can reasonably consider it "done" for my purposes, more than fine (or at least I'll have different considerations about breaking it). But to break it myself, now, in my particular circumstances just seems dumb.

There's more to it than wealth, but maybe this gives a better sense of what I mean by things will "change on their own". I don't necessarily mean for the better in any way other than it being easier to make a choice to focus on philosophy for a while.

And yes, maybe my reluctance is rationalization to hide the fact that I'd fail at learning philosophy if I actually tried. I don't think so, but that's possible too.

I think the best way to expose the rationalizations if they're there is to keep talking about details like I'm doing in this thread. Do you agree? Is there a better approach I'm missing?


AnonITA at 8:20 PM on January 11, 2017 | #8211 | reply | quote

> True, but I don't think I conveyed the nature of my situation correctly. There's lots of aspects but the easiest to describe is financial.

no that's not it. you misunderstood me. you wrote:

> I think another key consideration is I expect things to **change on their own** in the next few years for a variety of reasons. [emphasis added]

i replied to this, specifically, not to the general gist of what you were saying. that's why it was a single sentence quote instead of a big section quote or just a comment number.

none of your reply really has much to do with this.

i am aware you have the skill to fairly reliably continue on a similar life trajectory and e.g. get richer. but you applying you skill by performing skillful actions is not the same thing as things improving on their own. that's you actively earning more income, managing your finances, etc. doing your job is not something that just happens, it's something you do. just because you find something pretty easy to succeed at doesn't mean it happens on its own.


curi at 10:38 PM on January 11, 2017 | #8212 | reply | quote

Yes, I agree that earning more income, managing finances, doing my job, etc. will not happen automatically.

I intended to convey that with just my first word, "True", but evidently didn't. Or maybe you think I'm breezing by it too fast, treating it as a minor correction when you think it's major and deserves more discussion?

Anyway, the rest was about one problem I see with taking your suggestion of a 2 month focused effort on learning Popper / Rand.


AnonITA at 5:55 AM on January 12, 2017 | #8213 | reply | quote

> I intended to convey that with just my first word, "True", but evidently didn't. Or maybe you think I'm breezing by it too fast, treating it as a minor correction when you think it's major and deserves more discussion?

you wrote "True, but" which reads to me as your further comments are both related and contrary. like "ok i agree you're technically right, but..."

> Anyway, the rest was about one problem I see with taking your suggestion of a 2 month focused effort on learning Popper / Rand.

that wasn't clear. you began with "True, but"

i think it would have been clear if you said "True." and then you quoted something else and then wrote the rest as a second topic.


curi at 1:42 PM on January 12, 2017 | #8215 | reply | quote

> i think it would have been clear if you said "True." and then you quoted something else and then wrote the rest as a second topic.

OK.


AnonITA at 2:54 PM on January 12, 2017 | #8218 | reply | quote

Returning to one of the main issues as I see it:

> i think you can work on stuff in parallel and there will be connections. e.g. some idea about what's wrong with being a second hander will help give you an idea of where to look for firsthanded motivation (cuz you see what's going wrong there better, it gives you a clue about what to do instead).

Is there anything fairly specific in Rand (or elsewhere) I could read about what's wrong with being a second hander or where to look for firsthanded motivation?


AnonITA at 2:55 PM on January 12, 2017 | #8219 | reply | quote

> Is there anything fairly specific in Rand (or elsewhere) I could read about what's wrong with being a second hander or where to look for firsthanded motivation?

I'd say The Fountainhead is best. It describes well the impact of a life lived through the values of others rather and contrasts it to life lived through one's own values. Keating is the strongest example of second handedness, contrasted with Roark's selfishness. There are many many other examples of second-handedness.

Atlas Shrugged covers it too, but is much longer and covers a lot of other things. It's worth a read for it's own sake, but is not as focused on what you want.


SN at 3:09 PM on January 12, 2017 | #8220 | reply | quote

#8219

http://aynrandlexicon.com/lexicon/second-handers.html

Since I think you don't want to read a whole book, try her essays “The Nature of the Second-Hander” and “The Soul of an Individualist”!

In general the lexicon is great to find stuff like this.

This has some relevance: http://fallibleideas.com/lulie/initiative-and-responsibility


curi at 3:15 PM on January 12, 2017 | #8221 | reply | quote

Thanks for the essay suggestions. I read them.

One comment is that I seem much more motivated to read specific essays or sections related to a current question or problem than I am to read whole books.

I think you already knew that, but it is once again confirmed. I didn't have to "force myself" to read the essays, and I didn't get bored or want to take a nap while doing it.

Anyway, the first question I have from reading the Rand essays is about the relationship between second-handedness and altruism.

From "The Nature of the Second-Hander"

> After centuries of being pounded with the doctrine that altruism is the ultimate ideal, men have accepted it in the only way it could be accepted. By seeking self-esteem through others. By living second-hand.

I agree with the idea that being second-handed is the only way to accept altruism. My guess is it works in roughly the following way: You sacrifice your own values for the benefit of others, and your consolation prize is the esteem of those others. This is a mirage because sacrificing your values doesn't actually benefit them, and lots (most?) of the time they don't actually like you for it either.

But anyway, I wonder if the relationship also holds universally in reverse. Does being second-handed necessarily always involve some altruism? It's not readily apparent to me that it does.

Even though it's still second-handed, could I want the esteem of others for a reason other than altruism? Like in my own case: Could I want others to see me as successful because I want things from them that further my own values? Or is the only reason I could want to be seen as successful some kind of altruism?


AnonITA at 3:57 PM on January 13, 2017 | #8225 | reply | quote

> Even though it's still second-handed, could I want the esteem of others for a reason other than altruism? Like in my own case: Could I want others to see me as successful because I want things from them that further my own values? Or is the only reason I could want to be seen as successful some kind of altruism?

if you want their esteem for second handed reasons, then it's not about your own benefit. so, altruism.

i wait in line at stores instead of trying to go ahead of people to save time. this is in my interest. it's not about their esteem. trying to cut in line is more hassle than it's worth. i'm better off getting along with people regarding store lines. it isn't second handed to comply with the store's checkout line policy.

in the alternative: if there isn't a personal benefit, if it's just compliance for their benefit and not my own, then that'd be second-handed (caring what they think and want, not what I think and want) and sacrificial.

also, broadly, trying to suck up to people and be well-liked to get stuff from them is not an effective way to have a great life personally. the practical result is to sacrifice yourself and please them.


curi at 4:29 PM on January 13, 2017 | #8226 | reply | quote

> if you want their esteem for second handed reasons, then it's not about your own benefit. so, altruism.

and

> if there isn't a personal benefit, if it's just compliance for their benefit and not my own, then that'd be second-handed (caring what they think and want, not what I think and want) and sacrificial.

Both of those statements start with if. I'm trying to figure out whether they apply to my situation or not.

What I have determined in this thread so far is that I want certain other people to think that I'm a successful person. I don't yet know why I want that. I'm trying to figure out why I want it.

I don't think I want them to think I'm a successful person for their benefit. Some of them may benefit if I'm a successful person. But also maybe not. I can imagine ways of being successful these people wouldn't benefit from at all, but they'd still view as success, and I think would still fully address my concern about being seen as successful. I don't think I care whether they benefit or not.

That's why I am trying to figure out if second-handedness always implies altruism as (a / the) cause.

Cuz if there is altruism in my case, I don't know what it is and I would want to find out.

But if second-handedness can exist for reasons other than altruism, my guess would be that something other than altruism is the cause in my case. So I'd be looking to find what the other cause(s) are, instead of looking exclusively at altruism.


AnonITA at 6:56 PM on January 13, 2017 | #8227 | reply | quote

I just realized something might not be clear I'm assuming in the above:

I think that caring that certain people see me as a successful person is *definitionally* second-handed.

If that assumption is not true, I'd also want to know. If there's a way I could care that certain people see me as successful that's NOT second-handed, then I'd want to investigate whether I actually care for second-handed or first-handed reasons.


AnonITA at 7:00 PM on January 13, 2017 | #8228 | reply | quote

> Even though it's still second-handed, could I want the esteem of others for a reason other than altruism?

i addressed this question that uses second-handed as a premise and then asks if maybe not altruism. my replies then use second-handed as a premise. (if second-handed, then...)

so basically you asked an impersonal question and i answered it and then you're, without any discussion (or, I suspect, understanding) of my answer, trying to apply it personally and acting like it was intended to address your personal problems and those were actually what was being discussed.

> That's why I am trying to figure out if second-handedness always implies altruism as (a / the) cause.

which i literally just provided information about, which your reply does not engage.

note this question is again about what *implies* what -- logic -- not about you personally. which is what i addressed. then you disregarded what i said b/c it was impersonal. then re-ask the impersonal question.


Anonymous at 7:31 PM on January 13, 2017 | #8229 | reply | quote

> If that assumption is not true, I'd also want to know. If there's a way I could care that certain people see me as successful that's NOT second-handed, then I'd want to investigate whether I actually care for second-handed or first-handed reasons.

If you're a good person and they're good people, they will like you if they know you well. So if you judge someone as good, it makes sense to want them to like you, because if they don't like you it means there's been a misunderstanding that you should want to correct.

1) It might be they're not good, and so shouldn't like you

2) It might be you're not good, and so they shouldn't like you

3) It might be you're both good, but there was a mistake and they misjudged you and dislike you for false reasons

4) It might be you're both bad, then it's a bit of a crapshoot

Wanting the correct answer to be (3) is just being optimistic, it's a good result. (1) and (2) could still be good, if you fix the mistakes that makes someone bad.

But you need to be careful not to rose-tint your world and just assume (3) is good. Pursue the misunderstanding seriously. Because wrongly concluding that (3) is correct means at least one of you is bad and you're blinding yourself to it.

You also need to be wary of using something like this to rationalise second-handedness. Like you could be second-handed, but lie and say stuff like this in order to be liked by people who you think approve of it (or because you hate who you really are, and the lie helps you pretend to like yourself and be less unhappy).

One way of identifying if you're being second-handed about it is if your self-esteem changes depending on whether people like you or not. If you think your values are good, it wont make you feel bad if someone dislikes you for them. If you only pursue values for the approval of others, then you'll feel bad whenever someone disapproves of your supposed values.


SN at 4:54 AM on January 14, 2017 | #8230 | reply | quote

> But you need to be careful not to rose-tint your world and just assume (3) is good.

I mean "assume (3) is right."


SN at 4:58 AM on January 14, 2017 | #8231 | reply | quote

> If you're a good person and they're good people, they will like you if they know you well.

Some of the people know me well; some don't.

I think I care if even some people who don't know me well think I'm successful. One reason is because of how I think it will affect their behavior towards me. For example, what opportunities they'll let me know about.

There could be other reasons too, but I think that is one.

> So if you judge someone as good, it makes sense to want them to like you, because if they don't like you it means there's been a misunderstanding that you should want to correct.

I judge all of the people to be a mixture of good and bad. Regarding the specific context of success I'm analyzing here, I judge some of them as successful (explicitly: good at success - value producers) and others I judge as bad at success.

I think I care if even some people who I judge as bat at success think I'm successful. One reason is about winning other arguments I've had with them. For example, I think I'm acting as if my success can prove that God does not curse unbelievers. Or conversely, if I fail it proves them right about God cursing unbelievers. So: I avoid situations where visible failure is a significant risk.

Of course intellectually I know that my success or failure doesn't prove anything about God. But I think that's where some of the emotions about caring to be seen as successful are coming from.

As above, there might be other reasons too but I think that is one.

> One way of identifying if you're being second-handed about it is if your self-esteem changes depending on whether people like you or not. If you think your values are good, it wont make you feel bad if someone dislikes you for them. If you only pursue values for the approval of others, then you'll feel bad whenever someone disapproves of your supposed values.

I think my problem is primarily emotional. Intellectually I know that ex: pursuing a SENS lead with a high risk of failure can be a good value to pursue. But I'd feel bad about it because of what other people would think if it failed.


AnonITA at 7:50 AM on January 14, 2017 | #8232 | reply | quote

> so basically you asked an impersonal question and i answered it and then you're, without any discussion (or, I suspect, understanding) of my answer, trying to apply it personally and acting like it was intended to address your personal problems and those were actually what was being discussed.

I'm interested in my personal situation. But I also want the people discussing to find the discussion valuable, so they keep discussing.

One way I try to make the discussion valuable to those discussing is by talking about impersonal applications of the ideas involved.

If trying to de-personalize it is making the discussion worse (meaning: less valuable) I could stop trying to do that, and just speak directly about my situation.

Should I stop trying to de-personalize for now, or should I pursue getting better at de-personalizing then applying the results back to my personal situation before continuing on this thread?


AnonITA at 8:10 AM on January 14, 2017 | #8233 | reply | quote

>So: I avoid situations where visible failure is a significant risk.

this is one way you are living second-handed. you are letting them direct your life in some ways. you are letting them contaminate you and have a big impact on which types of situations you avoid.

and this IS sacrificial. you are sacrificing your mind and it's ability to direct your life.

you aren't using YOUR mind to judge which situations to engage in vs avoid. rather you are relying on others' conceptions of "success" to direct your life for you.

you are dependent on them for direction. you are eliminating many choices regarding what you could do with your life (many of which could be beneficial to you) because you are afraid those choices may result in some conventional people not thinking you are a success.

do you see how you are giving them power over you?

even though you have wealth and supposed "success", you are letting THEM direct YOUR life. but it's supposed to YOUR life, isn't it?

>For example, I think I'm acting as if my success can prove that God does not curse unbelievers.

and so one reason you are doing all of this (soul-destroying) sacrificing is because you are trying to prove a point about God to some dumb people?!


Kate at 8:58 AM on January 14, 2017 | #8234 | reply | quote

#8232 (AnonITA responding to me)

> I judge all of the people to be a mixture of good and bad. Regarding the specific context of success I'm analyzing here, I judge some of them as successful (explicitly: good at success - value producers) and others I judge as bad at success.

What sort of value? What do you think is good to produce? Like inventions? Ideas? Money? Art? Influence?

#8232 (AnonITA responding to me)

> I think my problem is primarily emotional. Intellectually I know that ex: pursuing a SENS lead with a high risk of failure can be a good value to pursue. But I'd feel bad about it because of what other people would think if it failed.

You shouldn't split your intellect from your emotion. They're all part of you. Your emotions are a result of your values.

Your emotional response means that you value the esteem of others more highly than doing something you claim to think is worthwhile. I suspect you're lying to yourself about thinking STEM is worthwhile. If you really think it's worth doing, you'd do it because you feel good about doing it regardless of how many people like you for it.

I guess your motive for saying it can be worthwhile is that if you *do* help and it succeeds, you think people will applaud you for helping with it. Ie second-handed motive.

#8234 (Kate responding to AnonITA)

>>For example, I think I'm acting as if my success can prove that God does not curse unbelievers.

>and so one reason you are doing all of this (soul-destroying) sacrificing is because you are trying to prove a point about God to some dumb people?!

Yeah. That's bad enough on it's own, but it's also mostly futile.

If you disprove someone who thinks on faith, they can make up a new explanation for why they're right. Like "god is in your heart even if you're blind to it" or "god works through you" or something.


SN at 10:51 AM on January 14, 2017 | #8235 | reply | quote

(OP)

> If I worked a 9-5 job, I would go to sleep by 8pm so I could wake up by 4am and do philosophy in the morning before work.

I've been trying this. Cool idea, it works.

I normally get a few hours of good focused thinking before I head to work. Then when I'm tired after work, I just do easy stuff like watching shows or doing chores.

Sleeping right after work is hard. I need some time to unwind and think about the day. I expect I'll need that less as I up my skill at the job and have less stuff I need to fix and less things I find stressful.

I pretty much don't socialise or try to catch stuff live, and I can handle any hands-on shopping after work. So I have very few things that interfere with this. On the rare days when I do, I can just juggle my sleeping/napping around it. At worse I'll have a bit of sleep debt to catch up on the next day.


SN at 11:04 AM on January 14, 2017 | #8236 | reply | quote

#8233 you should try to understand what i said. you haven't engaged with my explanation about a connection between altruism and second-handedness.


curi at 1:25 PM on January 14, 2017 | #8237 | reply | quote

#8230 the world isn't some friendly community of people who want to correct their mistakes. it's important to be more independent and less concerned about others. they generally aren't important. finding out about the ways each one is bad and misjudges you and your merits, and offering some corrections, and them learning nothing, won't get you anywhere.

and people aren't merely good or bad. people will e.g. be good about one or two things, average about a bunch of others, and bad about a bunch of others.


Anonymous at 2:00 PM on January 14, 2017 | #8238 | reply | quote

> I think I care if even some people who don't know me well think I'm successful. One reason is because of how I think it will affect their behavior towards me. For example, what opportunities they'll let me know about.

>

> There could be other reasons too, but I think that is one.

appeasing the gatekeepers of various opportunities is a way to *destroy your opportunity to have a good life*.


Anonymous at 2:02 PM on January 14, 2017 | #8239 | reply | quote

> I think I care if even some people who I judge as bat at success think I'm successful. One reason is about winning other arguments I've had with them. For example, I think I'm acting as if my success can prove that God does not curse unbelievers. Or conversely, if I fail it proves them right about God cursing unbelievers. So: I avoid situations where visible failure is a significant risk.

in other words, you're trying to teach them that even unbelievers live basically successful Christian lives that they approve of? you're trying to show these Christians, whom you disagree with, that they are more or less right about morality and their judgements of success are respectable.


Anonymous at 2:03 PM on January 14, 2017 | #8240 | reply | quote

> I think my problem is primarily emotional. Intellectually I know that ex: pursuing a SENS lead with a high risk of failure can be a good value to pursue. But I'd feel bad about it because of what other people would think if it failed.

and if they were even more confused about math and probability -- e.g. thought being a billionaire was less rich than a millionaire -- would you intentionally stop gaining any more wealth at like 900 million dollars?

is there a limit on what bullshit you'll pursue in *your* life according to *their* (mis)judgement?

is there any chance you're not actually as second-handed as you think and actually you partially agree with a lot of their typical views?


Anonymous at 2:07 PM on January 14, 2017 | #8241 | reply | quote

there are already plenty of examples of conventionally successful atheists without adding your name to the list. if anyone was going to change their mind about God because of this particular counter-example, they'd already have done it. unless they are a little kid, in which case you could tell them about about a few successful atheists that their parents neglected to mention.


Anonymous at 2:11 PM on January 14, 2017 | #8242 | reply | quote

> One comment is that I seem much more motivated to read specific essays or sections related to a current question or problem than I am to read whole books.

so find specific interests, then look up related essays/chapters/etc by good authors, and read them. do a succession of such things.

that's good and should be part of a normal life. i often do that (read targeted bits) in addition to reading whole books. it's easier to find things if you've read a book before (like you'll know what kinda stuff is in that book and where) but totally doable with books you haven't read.


curi at 5:37 PM on January 14, 2017 | #8243 | reply | quote

#8235

>I suspect you're lying to yourself about thinking STEM is worthwhile.

Oops. Bad mistake. I mean SENS.


SN at 8:48 PM on January 14, 2017 | #8244 | reply | quote

#8238

> the world isn't some friendly community of people who want to correct their mistakes. it's important to be more independent and less concerned about others. they generally aren't important. finding out about the ways each one is bad and misjudges you and your merits, and offering some corrections, and them learning nothing, won't get you anywhere.

I agree that not everyone is friendly and wants to correct mistakes. There are lots of people who are terrible or who don't want to change.

I wasn't talking about doing that with everyone in the world. You dropped context and didn't quote adequately.

If I learn nothing from it (ie they're not engaging/not correcting their mistakes/not offering a counter-argument) then I'll get somewhere. I'll find out it's not worth trying to do that again.

>and people aren't merely good or bad. people will e.g. be good about one or two things, average about a bunch of others, and bad about a bunch of others.

A lot of people are like that. They don't all have to be.

But yes, I'm approximating heavily by saying "good" and "bad" like this. I don't know exactly what AnonITA would consider "good" and "bad" anyway, I was speaking generally.

Speaking for myself I would mean something like:

good = seems to have potential, is not dangerous, wants to improve

bad = has dangerous ideas, is against error-correction, is a negative influence and has limited interest in being better


SN at 9:05 PM on January 14, 2017 | #8245 | reply | quote

> i wait in line at stores instead of trying to go ahead of people to save time. this is in my interest. it's not about their esteem. trying to cut in line is more hassle than it's worth. i'm better off getting along with people regarding store lines. it isn't second handed to comply with the store's checkout line policy.

It seems to me that the reason it's more hassle to cut in line than wait is because people will choose to create hassles for you if you cut in line. Right? If people didn't hassle you for cutting in line, and there was no clear statement from the store owner (something like "I don't want line cutters shopping in my store"), would it still be in your interest to wait rather than cut?


AnonITA at 5:41 PM on January 18, 2017 | #8255 | reply | quote

i don't think i get your point.


Anonymous at 6:31 PM on January 18, 2017 | #8256 | reply | quote

My point is to question a theory. Another way to ask:

I think the morality of waiting in checkout lines arises from the culture of the other people, not from the situation itself of having more customers wanting to check out than available checkers.

As another example, wanting to buy a drink from the bartender in a crowded bar is a similar situation to wanting to checkout in a crowded store. More customers than servers. But the culture in the bar is different. There's rarely any sort of line. It's kind of a social contest in terms of catching the bartender's attention without being too obnoxious. If you can do that and get served before someone who has been waiting longer, it's generally accepted. People rarely create hassles over it. And if they do, they hassle the bartender not the patron being served "early".

In such a case, if you have the skill to get served earlier it's in your interest to do so, right? Rather than i.e. pointing to someone who was there before you and saying "that guy's been waiting longer."

Is that right?


AnonITA at 7:27 AM on January 19, 2017 | #8257 | reply | quote

> Is that right?

i agree, but i don't know why you're saying this.


curi at 2:20 PM on January 19, 2017 | #8258 | reply | quote

The reason I asked is because of thinking about another statement from #8226:

> if there isn't a personal benefit, if it's just compliance for their benefit and not my own, then that'd be second-handed (caring what they think and want, not what I think and want) and sacrificial.

I get that your concern with waiting in line at the store is for your own benefit rather than for the benefit of others. I agree with that. The example of the bar was to confirm my understanding that it's not about personally benefitting from some kind of inherent fairness of lines, but benefitting from the way the other people react to your behavior.

I'm trying to take that a step further to consider what determines if I benefit from waiting or not? It seems to me that my benefit is determined by something that normally would imply second-handedness: what the other people in the situation think and want, not what I think and want. Because what they think and want is what determines how they react to my behavior.

Suppose I greatly prefer orderly lines like are common in stores to the social attention games that are common at bars as a means for dealing with the more-customers-than-servers problem.

And then let's consider a situation that could plausibly go either way: a guy behind a counter serving ice cream cones on a hot summer day. Sometimes, there's a line. And sometimes there's just a mass of people at the counter vying to be the next one the cone-server calls on. I've seen it both ways. Deli counters can sometimes be ambiguous that way too.

Seems to me that what it's in my interest to do with regard to waiting for ice cream or deli service depends on what the other people there seem to think and want. If they appear to think and want a mass at the counter with no line vying for attention, then even if I prefer lines it's not in my best interest to wait as if there were a line because I'll probably wait a lot longer. And vice-versa - if there's a clear line that people are respecting and observing and I cut straight to the counter, I'm liable to get hassled.

So the question I'm getting at: Is it second-handed to assess what the other people in a situation think and want, and then conform my behavior to those other people's expectations in order to minimize my combination of wait time + hassle? I don't think so, and this example was designed to illustrate precisely why. But I'm curious about your comments on this.


AnonITA at 3:26 PM on January 19, 2017 | #8259 | reply | quote

> So the question I'm getting at: Is it second-handed to assess what the other people in a situation think and want, and then conform my behavior to those other people's expectations in order to minimize my combination of wait time + hassle? I don't think so, and this example was designed to illustrate precisely why. But I'm curious about your comments on this.

the ordering system at bars you describe is awful and should downgrade one's opinion of bars substantially.

the waiting in line system at grocery stores is reasonable. and understanding that system isn't really about knowing about people. you could have an orderly line system where the other entities waiting are non-AI robots.

with the bar, there's social calibration skills involved in getting attention without overdoing it. nasty stuff. there's no way to have an equivalent scenario with non-AI robots for the other entities.

knowing minimal stuff about other people -- how to interact *impersonally* at stores, how to drive on the correct side of the road and obey traffic signs, how to navigate sidewalks without collisions or being arrested for public nudity -- is good.

getting involved in popularity contests, social calibration skills, caring what people think of you *personally*, trying to make yourself appealing to specific individuals (e.g. sucking up to your boss), trying to impress people and be well-liked ... this stuff is bad.


curi at 3:42 PM on January 19, 2017 | #8260 | reply | quote

> the ordering system at bars you describe is awful and should downgrade one's opinion of bars substantially.

I agree. I worded my preference for lines vs attention games as hypothetical, but it is also my actual preference.

But I don't think this addresses my question. Suppose it's a hot day, I want ice cream, and the first ice cream shop I walk into has customers massing at the counter, competing for the server's attention and the server doesn't seem to mind (1).

Is it second-handed to recognize what the other customers think and want in this situation, then decide I want ice cream and hate waiting enough to play the game they're playing as best I can?

Or should I stop wanting ice cream under those conditions? Walk 2 more blocks in the heat in hopes of finding a shop where the customers are lining up instead of massing? Attempt to convince the customers in the first stop to line up, or convince the server to demand it? All those options seem contrary to my self-interest, and I haven't thought of any others.

(1) As an aside, I think some servers prefer and knowingly allow the "mass at the counter" attention-competing system when there's a tip jar or other tipping mechanism where tips are not universal. By introducing an element of the server's choice about who to serve next into the serving process, it allows the server to assess who might be an especially good tipper, and give a recognizable favor by serving them "early" in hopes of being rewarded by them with a big tip. I don't know if this actually works to increase the server's tips or not, but it seems plausible.


AnonITA at 4:14 PM on January 19, 2017 | #8261 | reply | quote

> > the ordering system at bars you describe is awful and should downgrade one's opinion of bars substantially.

> I agree. I worded my preference for lines vs attention games as hypothetical, but it is also my actual preference.

> But I don't think this addresses my question.

it doesn't. it was a comment. other text i wrote addressed the question.

> Is it second-handed to recognize what the other customers think and want in this situation, then decide I want ice cream and hate waiting enough to play the game they're playing as best I can?

this is ambiguous.

what is "as best I can"? does it mean reading "How to win friends and influence people", and other stuff, and practicing it, and living and breathing it 24/7 so next time the ice cream situation comes up you're a fucking social master and can nail it?

there are various degrees of how much effort you put into social skills. i'd suggest you just do something on your phone and don't try super hard to mold yourself into a "charismatic" type person. it's not worth it. doing your best at this means destroying your soul. just let it go and accept non-great treatment in this minor case. don't learn how to be good at this.

i think you had something in mind more like doing your best in a *static* way -- your best in the moment with your pre-existing skills and no learning/changing. which is roughly OK, but can obscure the problem of having learned a lot more social skills *in the past* that's bad. and if you've already learned that stuff you should stop using it, even at the price of waiting more occasionally, because it'd be destroying you and has far more downside than upside.

> Or should I stop wanting ice cream under those conditions?

that's an option but another option is just to do a mediocre job of ordering ice cream. there are plenty of below average social-skill people in the crowd who get served. you don't have to do your best to get served here. you can just gradually make your way to the front and then spot a moment where you can say "hey i've been waiting 5 minutes already, please..." and he'll say sorry i'll get to you right after this next person i was about to serve.

or you could be more pushy, not in a socially skillful way, but just in an assertive don't-care-if-it's-rude way, which may work. it could even get you served just to get rid of you faster. shrug. a lot of people wait a long time because they are passive, shy, don't care about waiting, etc. it's totally possible to be assertive and loud just because you don't want to wait and aren't scared of people. this is different than being charismatic and keenly aware of social dynamics.

or if there's a tip jar you could just try to pay for service if you care that much by e.g. loudly asking for service when the server is looking and tipping at the same time so he sees the tip before you're served. and if you don't care that much, surely it's not worth a bunch of social crap. money is the cleaner and cheaper price.

one thing that'd make the tip work better is to give a socially acceptable excuse for why you need ice cream ASAP along with it. but getting good at that is the road i urge you NOT to go down. (i know this concept b/c i like understanding it abstractly and like high quality knowledge about whatever topic. but i just drew a blank on giving an example of a good excuse. which is totally fine with me. i don't care to actually do this or be any good at it.)


curi at 4:30 PM on January 19, 2017 | #8262 | reply | quote

> if you've already learned that stuff you should stop using it, even at the price of waiting more occasionally, because it'd be destroying you and has far more downside than upside.

How and why would it be destroying me?

If I've already learned the social skills required to wait less, what are the downsides to using them?


AnonITA at 5:53 AM on January 20, 2017 | #8263 | reply | quote

the social skills for the ice cream example are part of a package deal where people learn social stuff more generally. they aren't chosen individually.

the overall downside is spending most of your attention on secondhandedness, social games, etc, instead of solving or even thinking about your own problems and preferences.

ppl don't use social skill like chess skill. it's not an optional thing to turn on and off. it's an everpresent lifestyle which gets you worse results in many situations, requires being dishonest a lot (with yourself, not just with others), consumes tons of resources, and keeps you under constant large pressure to conform and never think outside the box.

whether any of this is how things *must* be doesn't matter. it's how they work today.


curi at 1:49 PM on January 20, 2017 | #8264 | reply | quote

> the social skills for the ice cream example are part of a package deal where people learn social stuff more generally. they aren't chosen individually.

OK, but I'm not considering whether to learn any more social stuff here, just whether to use what I already know. Even if it was a bad package deal to learn it in the first place, that seems like a different matter from whether it's a good idea to use it now.

> the overall downside is spending most of your attention on secondhandedness, social games, etc, instead of solving or even thinking about your own problems and preferences.

>

> ppl don't use social skill like chess skill. it's not an optional thing to turn on and off. it's an everpresent lifestyle which gets you worse results in many situations, requires being dishonest a lot (with yourself, not just with others), consumes tons of resources, and keeps you under constant large pressure to conform and never think outside the box.

This sounds kinda like the conventional idea of a avoiding a "bad habit". Like, it might not be bad to use social skill you already know in the ice cream shop as an isolated case, but doing that is commonly part of a lifestyle that includes using social skill lots of times when it is bad. So better to adopt a policy of avoiding it altogether, including the ice cream shop. Is that right?

Also, you seem to be speaking above as if the example we've been discussing (the ice cream shop) is secondhandedness. Do I understand that correctly?

If using social skills in the ice cream shop is secondhandedness, where is the altruism in that example? The problem I'm thinking about in that example is that I want ice cream, and I don't want to wait longer than necessary and don't want to get hassled. Those all seem like selfish motives rather than altruistic ones. The social games are a means to those ends. They may be bad for other reasons (like the "bad habit" reason) but I don't see where I'd be doing them to benefit others.


AnonITA at 5:28 PM on January 20, 2017 | #8266 | reply | quote

> OK, but I'm not considering whether to learn any more social stuff here, just whether to use what I already know. Even if it was a bad package deal to learn it in the first place, that seems like a different matter from whether it's a good idea to use it now.

the only realistic way out of that life is to reject the whole package, not to try to keep 1 foot in. it's hard enough to reject the social mindset in a principled, broad way where you wholeheartedly embrace some different attitudes. no one has ever succeeded in changing away from conventional social memes in a surgical pick-and-choose way.

your skills that apply to the ice cream scenario, and many other scenarios, are tied up (packaged) with immorality, altruism, second handedness, dishonesty, etc. that's how the memes work.

this is different than e.g. skill at chess which, even if it wasn't worth learning in the first place (and even if that learning process involved bad stuff like your parents forcing you to go to unwanted chess lessons), is a stand-alone skill that's no big deal to retain or use if you want to.

being good at social skills requires ongoing attention directed towards the social atmosphere. a good life requires redirecting that attention. continuing to do the ice cream thing socially is a setback in redirecting one's attention to good things. making a bunch of exceptions like that has ruined the overall self-improvement project for everyone who's done it.

could you get away with making ONE exception, rather than a bunch? in theory, sure. but why would you want to? no doubt if you try to improve you'll make far too many exceptions you're unaware of, by mistake. you really don't want to add any on purpose. adding any on purpose is incompatible with the serious-about-change and serious-about-rejecting-normal-social mindset needed to actually change.

> If using social skills in the ice cream shop is secondhandedness, where is the altruism in that example?

in spending a lot of your attention (both in the moment and in general) on understanding the social atmosphere and doing stuff to please others.

for example if you say a charismatic and socially calibrated excuse for why you need ice cream asap, that requires ongoing familiarity with what excuses stupid people accept and what behaviors they view as charismatic.

if you changed lifestyle to not have normal social stuff in it in general, you'd lose touch with what excuses would be good. you'd be out of date and forget things. (these things have shifting trends like fashion.)

and what clothes you're wearing affects how well you can do this. the right clothes will get you a different reaction for the same line. so this stuff is tied to other choices like wardrobe.

and your voice tone matters too. males often want a deep, firm voice. but it depends on the situation and what you're saying. females put even more effort into voice tones but the words for describing the nuances of their tones are worse and less well known.

being effective at social stuff involves lifestyle stuff. you're more effective if you live the life and like it. true believers are more convincing. and people wearing the right clothes as a matter of course, and talking in the right voice tones, and making the right amount of eye contact at the right times, and making the right sports references to the right recent sports game, and music and TV references, etc, etc, etc, are all part of social effectiveness and all have broad implications in one's life.

> The social games are a means to those ends. They may be bad for other reasons (like the "bad habit" reason) but I don't see where I'd be doing them to benefit others.

when the means to the selfish end is to sacrifice your life to others, that's altruism. a person better at promoting their self wouldn't spend a 999999 resources on secondhanded social for 50 selfish benefit.

and they wouldn't be interested in trying to get 4 benefit for 3 cost at risk of their soul if they miscalculate. Roark wouldn't and didn't do that. and a person trying to reform would do their best to reform, not try to cut corners. they'd be concerned they might fail to reform, rather than trying to optimize some tiny social stuff in some kind of calculated manner and get away with it. they'd try to cultivate an attitude of e.g. disgust towards social and not want to do it and care more about not sacrificing themselves in any way even a little than about much lesser issues like the ice cream thing.


curi at 5:53 PM on January 20, 2017 | #8267 | reply | quote

Is the conventional idea of a "bad habit" a good way to think about playing social games like in the example of the ice cream shop? Or are there significant flaws with the "bad habit" idea? Or is the "bad habit" idea fine but doesn't fit this particular case?


AnonITA at 7:10 AM on January 21, 2017 | #8268 | reply | quote

> a person better at promoting their self wouldn't spend a 999999 resources on secondhanded social for 50 selfish benefit.

> and they wouldn't be interested in trying to get 4 benefit for 3 cost at risk of their soul if they miscalculate.

Did you not mention what they would do with a ratio tilted heavily in favor of benefits from using social skill because you think that's super rare? Or some other reason?

Do you think situations where you can get something like 100 benefit for 3 resource cost on social skills you already have are reasonably common or super rare?


AnonITA at 7:21 AM on January 21, 2017 | #8269 | reply | quote

> Is the conventional idea of a "bad habit" a good way to think about playing social games like in the example of the ice cream shop?

i don't think so.

> Or are there significant flaws with the "bad habit" idea?

yes. the "habit" concept has similarities to the "addiction" concept which maybe you've seen me say stuff about already.


curi at 12:40 PM on January 21, 2017 | #8270 | reply | quote

> Did you not mention what they would do with a ratio tilted heavily in favor of benefits from using social skill because you think that's super rare? Or some other reason?

in the big picture, social is very expensive and the ice cream example is minor. (my first set of numbers)

if you try to look at it in an isolated way, someone might think there's only a little social involved here, and claim it's a minor gain with an even smaller loss (my second second of numbers). which is still a bad idea to pursue if one cares about the big picture. there are larger profits to pursue which don't get in the way of a bigger picture anti-social project.

> Do you think situations where you can get something like 100 benefit for 3 resource cost on social skills you already have are reasonably common or super rare?

uncommon. mildly rare.

if you look at an isolated scenario, e.g. a single salary negotiation, you can get stuff where the benefits of social shit look a lot higher than the resource costs (ignoring a bunch of pre-existing sunk costs). that one is tempting and debatable what to do even if one has an ongoing anti-social project. but getting faster ice cream shouldn't be a temptation to anyone who is trying not to be social in the bigger picture.

someone who doesn't want to be social could reasonably consider making an exception for a salary negotiation but not an ice cream purchase (unless you throw in another factor to raise the stakes like his child is crying over wanting ice cream ASAP).


curi at 12:49 PM on January 21, 2017 | #8271 | reply | quote

> the "habit" concept has similarities to the "addiction" concept which maybe you've seen me say stuff about already.

OK, I may need to revisit my thoughts on habits in general. Addiction has a chemical or at least non-idea-based component to the concept that I hadn't associated with habits at all. I thought of habits as nothing more than automated decisions, which you've forgotten or don't know how to easily change.

Do you know of a good concept other than "bad habits" that explains the general case of which second handedness and applying social skills is just one example?


AnonITA at 3:58 PM on January 21, 2017 | #8272 | reply | quote

"addiction" and "habit" are both negative words used to refer to things people do a lot of.

a non-negative word i use for substantially automated decisions is *policy*. i have pre-existing policies for how to handle a variety of situations which i often use instead of thinking up some new idea.

"policy" doesn't imply it being hard to change. lots of policies are pretty easy to change. you could use *entrenched policy* for one that's hard to change. "habit" can mean this, but it's a pretty ambiguous word which has other uses such as to stigmatize behaviors that someone doesn't want to change.

another common thing is people are accused of being *unable* to change in order to delegitimize their behavior, when actually they could change but don't want to.


curi at 4:11 PM on January 21, 2017 | #8273 | reply | quote

So would it make sense to say that applying social skills you already have in the ice cream shop example is generally a bad policy?

If so, is the main reason it is generally a bad policy that:

Such a policy is commonly part of a much larger set of entrenched policies about social skills. And that much larger set of entrenched policies has a very negative impact on your life. And it's very difficult to separate out just the social skill policies that don't have a negative impact on your life. So you're much better off trying to eliminate the whole set of entrenched policies rather than trying to pick out and salvage the few that might be OK.

Is that a good way to think of the ice cream shop example?


AnonITA at 4:27 PM on January 21, 2017 | #8274 | reply | quote

> So would it make sense to say that applying social skills you already have in the ice cream shop example is generally a bad policy?

applying significant social skills in this example, and broadly in general, is a bad idea *if* you have a serious project of stopping being social.

> Such a policy is commonly part of a much larger set of entrenched policies about social skills. And that much larger set of entrenched policies has a very negative impact on your life. And it's very difficult to separate out just the social skill policies that don't have a negative impact on your life. So you're much better off trying to eliminate the whole set of entrenched policies rather than trying to pick out and salvage the few that might be OK.

yes. but you have to do some separation or thinking about which social stuff is bad and waht's ok. driving on the right side of the road is a social cooperation thing. wearing clothes in public is a social cooperation thing. not disturbing the neighbors with your blender at 3am is a social cooperation thing. saying "hello" when you answer a phone is a social convention thing. these are reasonable and practical and don't involve stuff like social climbing, popularity contests, being very attuned to body language, etc


curi at 4:47 PM on January 21, 2017 | #8275 | reply | quote

> applying significant social skills in this example, and broadly in general, is a bad idea *if* you have a serious project of stopping being social.

Why would someone who already has social skills want to undertake a serious project of stopping being social? What are the pros and cons of such a project?


AnonITA at 7:08 PM on January 21, 2017 | #8276 | reply | quote

pros: get their soul/self back

cons: harder to suck up to a bad boss to keep a shitty job, harder to marry or stay married to a bad person


Anonymous at 7:18 PM on January 21, 2017 | #8277 | reply | quote

social junk

> Why would someone who already has social skills want to undertake a serious project of stopping being social? What are the pros and cons of such a project?

pros: don't waste time on boring shit, do more stuff that is good and valuable, don't say stuff you don't mean.

cons: people who like wasting time on boring shit will be less inclined to cooperate with you.


oh my god it's turpentine at 6:28 AM on January 22, 2017 | #8279 | reply | quote

>Why would someone who already has social skills want to undertake a serious project of stopping being social?

I think if you get down into the details enough the question almost answers itself.

E.g. Why would a woman who already has skill at makeup and pleasing conventional guys via flirting etc., want to undertake the project of not dedicating a huge portion of her life to catering to static memes and irrational whims?


Extremely Mysterious Person at 12:26 PM on January 22, 2017 | #8282 | reply | quote

Makeup and pleasing conventional guys is a useful example.

I want to ask a question to calibrate how much disagreement I have about social stuff with FI.

Thinking of the makeup and pleasing conventional guys example of social stuff, and regarding whether to undertake a serious project of stopping being social, do you think "the question almost answers itself" [with a yes] for Melania Trump?


AnonITA at 6:00 AM on January 23, 2017 | #8283 | reply | quote

I don't know why you picked this example. It seems really exceptional. So it doesn't compare well to most people's situations.

Melania Trump has made major commitments involving, among other things, dressing well and having a certain appearance. She'd be breaking important promises if she stopped.

This isn't just mundane stuff like she'd disappoint her husband who isn't allowed to fuck anyone else and who likes makeup. She's First Lady. She promised America she'd be a certain kind of person. She's a representative for our country who has projected a certain image and is reasonably expected to continue to do so. She shouldn't go back on those election campaign promises now that she's been trusted with major political status.


curi at 2:39 PM on January 23, 2017 | #8284 | reply | quote

> I don't know why you picked this example. It seems really exceptional. So it doesn't compare well to most people's situations.

Yes, it was the most exceptional example I could think of. Worldwide there's maybe 100 people in her type of position (if that). That's why I said I was using it to calibrate how much disagreement I have with FI about social stuff.

I don't have a high opinion of the importance of social stuff compared to most people. But I get the strong sense I have a higher opinion of it than you.

If you'd said something like that even Melania should undertake a serious project of stopping being social right away, then I'd know something important about how much we disagree, and we could discuss why.

Instead, I agree with your response about Melania.

So next I will try to back off how exceptional the examples are until I find one where we disagree, and then we can discuss that.

If we get very far before encountering a disagreement I'll need to switch to hypothetical descriptions instead of actual people. But I think an example of the next step down the ladder for calibration is pretty easy. Still keeping with the makeup and pleasing conventional guys example of social stuff how about Kim Kardishian?

I picked Kim as an example of someone who has built a life around doing the makeup and conventional guy appeal social stuff. I would guess there are thousands of people like that worldwide. Most not as successful at it as Kim is, but plenty who are successful enough that it is approximately their whole career.

Do you think it'd be a good idea for Kim Kardashian to undertake a serious project of stopping being social?


AnonITA at 3:16 PM on January 23, 2017 | #8285 | reply | quote

ok. btw high public offices really are a sacrifice in some significant ways.

> Still keeping with the makeup and pleasing conventional guys example of social stuff how about Kim Kardishian?

(i know almost nothing about her)

she should change for sure. easy case. she has plenty of money and can retire (more specifically, she can make a transition without worrying about making any money during the transition. she can take as long as she needs to learn new skills, a new lifestyle, etc, without being under time pressure from money). she has nothing of value to gain from continuing what she does now. she could make more money, but choosing a soul/self over money is such an easy choice when you're already super rich. additional dollars won't let her buy anything personally important that she can't buy now (the best current use of additional dollars for her -- and a lot of her current dollars -- is stuff like donating it to the David Horowitz Freedom Center. if she became a great person then she could find some better uses but that's no reason to stay a bad person to make more money). the other stuff she could lose is her social status, popularity, dumb friends, etc. that stuff isn't appealing/tempting if one wants to start thinking and being productive and other good stuff.

her lifestyle is awful. she has a bad life. her life sucks. i wouldn't want to be her. if i were in her shoes, of course i'd immediately try to drastically change my life. some fans will be disappointed by so what. if she managed to become a good person it'd actually help her best and most worthy fans.


curi at 3:31 PM on January 23, 2017 | #8286 | reply | quote

OK good, I think we disagree. I don't think she's an easy case.

I don't know a bunch about her either. I definitely don't know enough to assume:

> she can make a transition without worrying about making any money during the transition.

I don't know for sure, but my best guess is that's false.

She's definitely rich in the sense of having a high current income. But I have no idea what her balance sheet or cash flow statement looks like. And those things, rather than her current income, are what matters to whether you have to worry about making money during a transition.

She could have debts in excess of her assets, or high expenses that aren't straightforward to cut, such that if she stopped earning a high income she'd soon be destitute. That kind of financial situation is fairly common among celebrities. Most of them aren't good at saving or investing and lack even basic frugality skills.

Also, lots of her assets might be contingent - royalty rights on stuff that depend on her continued popularity for example. She could have a very high net worth on paper that is mostly or entirely dependent on keeping her current lifestyle going. That's very different from having a high net worth that's invested in stuff like stocks and bonds.

She could also have ongoing contractual obligations where social stuff is expressly required or at least reasonably implied. She could be found in breech of those contracts and have to pay lots of penalties if she stopped being social. So even if she has a high net worth in stuff that doesn't depend on her ongoing celebrity, she could lose it all in a judgment for willful breech of contract.

Maybe you know more about her situation than I do and know that none of those things are a significant problem for her. I don't, and I think the safest assumption is that at least some of them are.

So I think that before she should undertake a serious project of stopping being social, she would need to undertake a serious project of preparing for a transition, including things like:

- Know what her assets and debts are, how they balance and what they depend on

- Know what her expenses are, and what she could reasonably do to reduce expenses given her actual skill set, and what her expenses can reasonably be expected to be in the future

- Learn about investing and money management so she'd know what to expect from her investments while she's not earning an income

- Make sure she had a great many years of expenses very safely invested in things that aren't contingent on her being social

- Make sure she wouldn't run into contractual problems by stopping being social, or being fully prepared for the problems she could reasonably expect

Basically, I think she'd need to figure out a lot and possibly put a lot of stuff in place for her "post-social" life *before* undertaking a serious project of stopping being social.

Do you know the problems I mentioned aren't relevant to her situation, or do you think she's an easy case despite any of those problems?


AnonITA at 4:19 PM on January 23, 2017 | #8287 | reply | quote

i googled before posting and it said she has net worth of $150 million. that means massive assets in excess of debts. i relied on this information for my post. did you make no attempt to look this up, or are you disputing this without saying anything to directly address it (just general knowledge that some people with high salaries aren't as rich as you think and knowing a few pitfalls they run into), or what?

> Most of them aren't good at saving or investing and lack even basic frugality skills.

all the more reason to start becoming a good person ASAP while she's still ultra rich.

also from googling more when you disputed her net worth, she is apparently an entrepreneur who has made tens of millions of dollars with stuff related to her image/brand like a mobile game. so either she has some competence at some of this stuff, or has hired some decent help. you could have found this out with very minimal research. i don't think it's reasonable to choose the example, do zero research, and contradict the common view of the matter because of some general knowledge that some celebs ain't super financially well off.

anyway i don't see much point in debating how rich she is. can we simply assume the $150 million net worth is legit and move on? if you want an example where the person isn't already super rich please pick someone who isn't commonly believed to have a $150 million net worth...


curi at 4:42 PM on January 23, 2017 | #8288 | reply | quote

> i don't think it's reasonable to choose the example, do zero research, and contradict the common view of the matter because of some general knowledge that some celebs ain't super financially well off.

OK Fair. I didn't do any research.

I read your position as a guess that all the problems I mentioned (which aren't limited to just net worth) aren't relevant to Kim's situation. Even if that wasn't your intent, I have no specific reason to dispute such a guess for Kim. And no interest in doing research that might uncover such reasons.

But I think it's more interesting to consider for discussion sake what if one of the problems did apply. Suppose for example that most of Kim's assets were contingent. Stuff like the calculated net present value of businesses that depend on her ongoing popularity for their solvancy. Without those assets, her net worth would be negative.

If that were the case, what then? Would you still recommend that she undertake a serious project of stopping being social?

Another way of asking what I'm interested in would be to consider two plans for Kim:

Plan A:

1. Tryhard to stop being social.

2. Tryhard to figure out how to live well without being social. Maybe go bankrupt during the transition.

3. Rebuild a good life without social stuff from wherever you end up.

Plan B:

1. Tryhard to figure out how to live well without being social.

2. Put all the pieces you think a non-social life will require in place.

3. Tryhard to stop being social.

4. Live a good life without social stuff.

I am much inclined toward Plan B, but I suspect you might favor Plan A or at least something substantially different from Plan B. Do you?


AnonITA at 4:19 PM on January 26, 2017 | #8293 | reply | quote

why do you want to discuss this example instead of just using a regular non-rich person who has neither large assets nor large debts and isn't obligated by any major business contracts?

you want to hear that e.g. maybe someone who makes 50 million a year, but has 100 million of debts, ought to keep their job for several more years? i guess. but the same thing doesn't apply with smaller numbers. e.g. you totally don't stay chained to a 50k/year job just b/c you have 100k debt.

big picture your analysis neglects various important things like:

you can't just figure everything out as one step, then do it as another step. basically you have to learn a bit, start living it, adjust a bit, learn a bit more, etc.

and you can't *want* to stop being social, in a serious way, before you know a lot about this stuff. in plan B you basically put the part with all the motivation as step 3.

and you can't know what a non-social life will require until you actually create one. your stuff is out of order.

basically the plans you list are both naive, simplistic and terrible. they have nothing to do with incremental progress. they have nothing to do with a learning progression. they are formulated much more like "ok so do i need to take 2 or 3 university classes? i know the order to take them in, i just don't know if i can skip one of the classes or not."


curi at 4:42 PM on January 26, 2017 | #8294 | reply | quote

> pros: get their soul/self back

I'm having trouble tying this back to reality.

Can you give an example of something concrete and valuable that someone who got "their soul/self back" would be able to get/do, that someone who was still being social wouldn't?


AnonITA at 4:42 PM on January 26, 2017 | #8295 | reply | quote

> why do you want to discuss this example instead of just using a regular non-rich person who has neither large assets nor large debts and isn't obligated by any major business contracts?

I think my situation falls somewhere between a rich person and a regular non-rich person. I have a better life than a regular person. I have more to lose than they do, though not as much as someone like Kim.

> you totally don't stay chained to a 50k/year job just b/c you have 100k debt.

Lots of people totally do...especially if they have a family to support. Maybe they shouldn't. But if that's true, I don't understand why they shouldn't.

Consider the guy with a 50k/year job that requires being social, 100k in debt, with a kid. What else would I tell such a person other than to keep working the job until the debt is paid off and he's got a bunch of money in the bank, OR his kid is grown and self sufficient? Sure there's marginal things he should do like earn and ask for a raise, look for a job that pays better, start a side hustle. Those are fine and good but don't alter the fundamental advice to basically keep doing what he's doing, including the social. I don't know of any other responsible advice to give.

> you can't just figure everything out as one step, then do it as another step. basically you have to learn a bit, start living it, adjust a bit, learn a bit more, etc.

If "start living it" is majorly disruptive to an ongoing good life, then how do you deal with that in an incremental way?


AnonITA at 5:12 PM on January 26, 2017 | #8296 | reply | quote

> Lots of people totally do...especially if they have a family to support. Maybe they shouldn't. But if that's true, I don't understand why they shouldn't.

yeah people do, but shouldn't.

among other things, there are a million other jobs paying 50k/yr or even say 70k/yr so you don't need to be chained to one particular 50k job you hate. there are better alternatives with significant general accessibility (in e.g. the US, not Somalia). but if you make 50 million then there aren't other accessible jobs with anywhere near the same pay.

> Consider the guy with a 50k/year job that requires being social, 100k in debt, with a kid. What else would I tell such a person other than to keep working the job until the debt is paid off and he's got a bunch of money in the bank, OR his kid is grown and self sufficient? Sure there's marginal things he should do like earn and ask for a raise, look for a job that pays better, start a side hustle. Those are fine and good but don't alter the fundamental advice to basically keep doing what he's doing, including the social. I don't know of any other responsible advice to give.

or he could get another 50k/year job that doesn't require being social. dunno why you considered changing jobs to get a marginal pay increase but not switching to a non-social job.

> If "start living it" is majorly disruptive to an ongoing good life, then how do you deal with that in an incremental way?

start be recognizing *problems* with this "ongoing good life". then start doing *problem solving*. all life is problem solving. and problems are soluble.

by phrasing the life as "good" rather than *problematic*, you are obscuring the mechanism of progress.


curi at 5:18 PM on January 26, 2017 | #8297 | reply | quote

> Can you give an example of something concrete and valuable that someone who got "their soul/self back" would be able to get/do, that someone who was still being social wouldn't?

Can you think of anything concrete and valuable Roark got to get/do that Keating didn't?


Anonymous at 7:16 PM on January 26, 2017 | #8298 | reply | quote

#8295

>Can you give an example of something concrete and valuable that someone who got "their soul/self back" would be able to get/do, that someone who was still being social wouldn't?

Say you have a great new idea for how to do something. But it contradicts how your social group thinks.

Since this idea isn't approved of by your social group, you'll lose status if you act on it.

So you're trapped. You can't try new ideas but also maintain your social status.

You could pretend to still agree with them and act on your idea in private, but lying effectively is hard work and will take a ton of your time. Plus if your new idea contradicts your social group, you'll almost certainly have to act on their bad ideas while socialising, so wasting more of your time doing stuff wrong.


SN at 7:54 PM on January 26, 2017 | #8300 | reply | quote

soul

> Can you give an example of something concrete and valuable that someone who got "their soul/self back" would be able to get/do, that someone who was still being social wouldn't?

You would know about stuff you want to do and do it. This is more interesting and fun.

You don't waste time and money on boring shit, e.g. - night clubs.

You don't pretend a person is right when he sez something wrong. You're also clearer about which ideas are good and which are bad. Judgements you can never act on or express are not much use.


oh my god it's turpentine at 10:39 PM on January 26, 2017 | #8301 | reply | quote

>> pros: get their soul/self back

>I'm having trouble tying this back to reality.

>Can you give an example of something concrete and valuable that someone who got "their soul/self back" would be able to get/do, that someone who was still being social wouldn't?

they'd be able to direct their own life.

they wouldn't avoid situations (such as a SENS research opportunity or whatever example you want to put in there) because they are afraid of being classified as "failure" in other people's eyes.

they don't depend on others for their conceptions of success and failure. this means also that they haven't *internalized* others' conceptions of success and failure.

there are other concrete examples, too. like lots of people get married, go to college, become an architect (Keating) all because they are looking to others to direct their lives.

if you get your soul/self back, you stop making these types of mistakes.


Kate at 9:33 AM on January 27, 2017 | #8302 | reply | quote

Non-social jobs

> or he could get another 50k/year job that doesn't require being social.

I am trying to think about "doesn't require being social" to a high standard, which I assume you were too. To a low standard, my current job doesn't require being social. There may be somewhat "less social" jobs still, but that's not what I'm thinking about / interested in atm.

I am also thinking of the actual requirements of both *getting* and *keeping* a job, not just what is explicitly written down or stated by the employer about it.

I am also thinking about a "job" in the sense of something that provides a relatively reliable income stream for a reasonable time commitment. Contract or freelance work qualifies, but only if someone of reasonable skill could expect to clear ~50k doing it in most years by working (or trying to find new work) ~40 hours/week.

In that context, I think "doesn't require being social" to a high standard includes meeting all of:

- Doesn't require physical presence or "visibility" (video or pictures) beyond the actual, objective requirements of the job if any.

- Doesn't require you to live in any particular city or location, beyond the actual, objective requirements of the job if any.

- Doesn't require dress, mannerisms, or language restrictions beyond ordinary public standards.

- Doesn't require any arbitrary credentials (like a degree from a school that's socially considered "good"), just the knowledge and skill to do the job.

- Doesn't require a set number of work hours or synchronous schedules beyond the actual, objective requirements of the job if any.

- Doesn't require producing things for solely or mostly social reasons, like pretty reports or proposals or other busywork. "Just the facts" kind of reports on objectively valuable data are the only kind ever required.

- Doesn't require applied social skill to obtain expeditious approval or other things you need to do the job you were hired to do (like access to a database, or assistance from another employee with a particular expertise).

- Doesn't require displaying interest in "advancing" into roles that do require social stuff.

- Doesn't require acting in socially deferential ways to bosses / others in authority / customers.

- Doesn't require "softening", spinning, "sandwiching", or otherwise downplaying and de-emphasizing criticisms, disagreements or problems with anyone.

- Doesn't require crafting descriptions of your accomplishments in socially calibrated ways that inflate the quantity/quality/value in order for your work to be fairly evaluated and compensated.

- Doesn't require teaching or directing anyone in socially calibrated ways.

- Doesn't require engaging in non-work activities with anyone.

- Doesn't require soliciting or interacting with customers or suppliers in socially calibrated, rather than objectively optimal, ways.

There's probably more I'm not thinking of. But I think the above list is quite enough to eliminate all of the 50k/year jobs I can think of that would be available to a regular non-rich person who's 100k in debt and has a kid.

Maybe I'm being too strict? Is there stuff in my list that isn't actually social?

Assuming I'm not being too strict, I sincerely hope that I'm just ill informed about the available jobs. If I am, and there are lots of available jobs that don't require being social in all the ways I listed, I'd be glad to know what they are.

Are there? Can anyone give one, or better yet three, actual examples that would stand up to FI-level scrutiny?


AnonITA at 10:29 AM on January 27, 2017 | #8303 | reply | quote

we were talking about celebs whose life is social. there are tons of jobs which don't involve life-ruining social.


Anonymous at 10:42 AM on January 27, 2017 | #8304 | reply | quote

> we were talking about celebs whose life is social.

I thought we'd left the celeb scenario.

curi asked:

> why do you want to discuss this example instead of just using a regular non-rich person who has neither large assets nor large debts and isn't obligated by any major business contracts?

...and then mentioned the 50k/year with 100k debt scenario.

I answered curi's question, and then started talking about the 50k/year scenario. I wasn't thinking about celebs when I wrote my last comment at all.

Regardless:

What is the difference between "life-ruining" social and not-life-ruining social?

Which of the social things on the list in my last comment are life-ruining if a job requires them, and which not?


AnonITA at 11:20 AM on January 27, 2017 | #8305 | reply | quote

> Can you think of anything concrete and valuable Roark got to get/do that Keating didn't?

Work in a rock quarry?

In case you think so, I'm not being facetious. It's actually relevant to one of my concerns. Which is, needing to do a job that's menial and low paid because I gave up my current position which is neither.


AnonITA at 11:25 AM on January 27, 2017 | #8306 | reply | quote

> start be recognizing *problems* with this "ongoing good life". then start doing *problem solving*. all life is problem solving. and problems are soluble.

> by phrasing the life as "good" rather than *problematic*, you are obscuring the mechanism of progress.

I think the discussion is going in circles and I'm not sure how to resolve that. I'll try to summarize at a high level what I think is going on and then see if you have any suggestions how to proceed.

Yes, I agree problems are inevitable and soluble. And I agree that I have problems in my current life.

But I also judge my current life to be good, and I think that judgment matters a lot. If I hated my current life, it'd be a different situation. If I thought I had little to lose, it'd be a different situation.

The problem that I'm interested in solving is that I'm likely to die a lot sooner than I want to. A big reason I'm interested in the dying problem is precisely because I judge my life to be good. I don't want it to be over.

Thinking about ways to work on the dying problem all seem to involve creating more urgent problems in my life though.

For example, I can think of ways I could have some some significant positive impact on SENS, but I also know those ways would create a money problem in my life that I don't currently have.

Would I rather have the money problem than the dying problem? Absolutely - if it was a sure thing. Or even if it was a high probability. But it's not. And I don't think I prefer the certainty of a money problem + small chance of helping SENS to my current life's problem state.

In discussing that situation, I discovered that I care more about social stuff than I previously thought. That (social) problem doesn't bother me much on its own; I am interested in it because of how it affects decisions like whether and how to work on SENS.

But still, the same pattern seems to be present with the second (social) problem as with the first (dying) problem. I can think of ways that I could make progress on the social problem, but I also know those ways would create specific other problems in my life that I don't currently have.

Would I rather have the other problems than the social problem? I don't think so - my first guess is no; I prefer my current problem situation.

Point being: problems are inevitable, but knowingly creating avoidable ones is not. I'm trying to get to a life with problems that I judge are better than the problems that I currently have.

I don't know of any way to actually do that. I can think of lots of ways I could have *different* problems, but none that I judge to be *better*.

Do you have any suggestions for how to proceed from here?


AnonITA at 12:15 PM on January 27, 2017 | #8307 | reply | quote

> I also judge my current life to be good

how can it be good when you haven't learned much good philosophy and you still have lots of conventional ideas?

it doesn't make sense that this fact scenario leads to a "good life" by objective standards. maybe it leads to a good life by conventional standards, but not objective standards.

having low standards could also explain why you can't seem to identify any problems / pain points in your life which you think are worth working on.

(since you don't think the dying problem nor the not-being-social problem are good problems to work on.)

like do you ever feel any unwanted emotions? are you interested in learning *why* you lack motivation for some things (where you currently don't know why)? have you ever had a discussion with someone which you think you could have done better? do you want to learn how to have better relationships and problem-solving processes with the people in your life? all this stuff perfect already?

is there *anything* you want to work on changing for the better?

or is there always a justification for why you don't want to work on the problems which you do identify? e.g. it'll create too many unwanted new problems, fear of fast and disruptive change, maybe privacy issues idk, etc.

also, what has philosophy helped you with so far?


Kate at 2:18 PM on January 27, 2017 | #8308 | reply | quote

http://curi.us/1945-scheduling-and-thinking#c8190

AnonITA wrote:

> I think being more intellectual than normal is a component of the particular image that I want to have with the groups that I care about. For example, I can say things that people in those groups will find surprising and "profound" in a good way - stuff they don't hear from anyone else. I do it often enough that they notice, remember it about me, and will sometimes comment positively about it. At the very least, FI gives a good return on that kind of skill per unit of effort compared to other "intellectual" things that I could do or did before FI.

> But I also think I'm at the point of diminishing returns. If I increase my rate of "profound" statements much I think my image would go from "the successful guy who says some of the most interesting things," to "that egghead who lets intellectual naval gazing ruin his true potential." It's a fine line; I think I've recognized it at some points in the past and backed off saying things I'd otherwise say.

BIG HUGE :(

> To be clear, I'm not saying that I think image is my only motivation to participate in FI. Or that it's my primary motivation. It could be 5% or 95%. At this point I don't know how much it is.

>But however much it is, I don't want to disturb it until I have something effective to replace it with. Lots of people quit FI. I think that could be a real danger for me too.

i don’t understand this. you don’t want to disturb your motivation? why does solving the problem of figuring out your purpose in participating on FI first require having something effective to replace FI with? can you clarify?

it sorta sounds like you are saying, “i admit i’m acting aimlessly here. i don’t know why i’m doing what i’m doing wrt being on FI. but i don’t want to do anything (such as think about my purpose?!), which might put me at risk of quitting FI while not having an effective replacement.”

how helpful do you think the motions of FI participation are going to be (in helping you solve *good* problems) if you are approaching FI aimlessly and without a clear purpose?

also, does what you wrote above concern you in a big way? any thoughts such as, “oh fuck. I spend a decent amount of time participating with the FI community, yet I now realize that I don’t even know why I’m doing that!!! am I this aimless with other activities I engage with? this mindlessness is a disaster that i need to urgently try to fix. how am i going to catch all of the mistakes i make with a mental approach like this?”


Kate at 6:23 PM on January 27, 2017 | #8309 | reply | quote

>> I also judge my current life to be good

> how can it be good when you haven't learned much good philosophy and you still have lots of conventional ideas?

I think this would take a very long discussion to cover thoroughly.

One short comment: I think "good" is more appropriate than other short and simple terms of evaluation I could choose like "awful", "bad", "sucks", "mediocre", or...on the other side... "great" or "perfect".

I have criticisms of all such terms of evaluation as comprehensive descriptions - they're not. But given that they're not, and the need to convey my evaluation quickly and easily in a discussion like this, I have no criticisms of using the evaluation "good" for my life in this context.

I'm not interested in convincing you that my life is good though. Feel free to judge that some other term of evaluation better fits my life.

> having low standards could also explain why you can't seem to identify any problems / pain points in your life which you think are worth working on.

You mischaracterized my situation.

I work on problems constantly. I actually think most of them are worth working on. There are a few that are like dumb bureaucratic shit at work. But I mostly like what I do.

The specific situation I'm discussing here is making major changes in life direction. There are problems / pain points in that area, that the only ways I know of working on currently would seem to create *less preferred* problems than the ones I already have.

In such a case it seems to me that just "working on" those problems would be dumb. I need to find ways to work on them that get me to a problem state I prefer.

One way that could happen is to change preferences so that I prefer the problem set that working on them will create.

Another way would be to think of some other way of working on the problems that don't cause the problems I don't prefer to have.


AnonITA at 5:48 PM on January 29, 2017 | #8310 | reply | quote

>> how can it be good when you haven't learned much good philosophy and you still have lots of conventional ideas?

>I think this would take a very long discussion to cover thoroughly.

So what?

You're giving up?

> I'm not interested in convincing you that my life is good though.

You're focusing on social stuff. Your social problem is getting in the way of progress again. You're thinking about getting approval rather than aiming for a better life.

Are you interested in understanding why people (who you claim you learn good ideas from) think your life isn't good? Are you interested in understanding ways your life could improve?

It doesn't seem like it.

What *are* you here for?

You've said you just do it for social status, and you actually want to stop improving for social status too.

Seems like you're going to quit FI anyway because you're good enough to impress your social group and you don't want to be any better than that because you think they don't like smart people.

Maybe some part of you already knows that and that's why you're worried about disrupting your motive.

#8309 highlighted that too but you evaded it

Why are you evading?


SN at 8:04 PM on January 29, 2017 | #8311 | reply | quote

it doesn't make sense to call one's life as a whole good or bad. it's at the *beginning of infinity*. it's infinitely problematic. that doesn't mean bad. both "good" and "bad" are misleading characterizations. it's *problematic* (aka open to improvement). and if one understands a problem then one ought to care to improve.

anyone who really believes they are at the *beginning of infinity* would have no interest in resting on their laurels or settling down to preserve the status quo without substantial concern for progress. and anyone who doesn't believe they are at the beginning of infinity – who thinks 989234 is a large amount of progress and near utopia -- is mistaken.

some people are broadly aware of social as a problem but are still involved in it, so they want to improve that. so they'd e.g. be wary of intentionally engaging in social calibration to get ice cream marginally faster because it conflicts with their ongoing project.

some people don't understand much about how and why social is a problem for them. they ought to improve another problem they know about or learn more about their problems.


curi at 11:15 PM on January 29, 2017 | #8312 | reply | quote

> You're giving up?

No. I said in #8307:

> I think the discussion is going in circles and I'm not sure how to resolve that.

I still don't think I've resolved it, so I'm trying other approaches.


AnonITA at 5:38 PM on January 30, 2017 | #8313 | reply | quote

> it doesn't make sense to call one's life as a whole good or bad.

I agree in some contexts and disagree in other contexts. In the context of this discussion I disagree.

I agree that all lives are problematic.

I agree that we're at the beginning of infinity.

Referring to your number, I don't think being at the beginning of infinity conflicts with the value of calling 989234 "large" or "good" and 107 "small" or "bad" in some contexts.

I don't expect you actually disagree with that, though if you do please say so. What I expect is you disagree about whether this discussion is such a context.

> both "good" and "bad" are misleading characterizations. it's *problematic* (aka open to improvement).

Rand's life, Trump's life, Hitler's life, my life, and a crack whore's life are (or were) all problematic. True, but misses the point. Some people have a lot more to lose than others. They have a lot more to lose because they've done more good things in their lives than people with less to lose.

I think having more to lose matters to what problems they should work on solving and how they should go about that work in terms of things like risk and starting over.


AnonITA at 5:43 PM on January 30, 2017 | #8314 | reply | quote

there are distinctions between lives that are important here. calling some "good" and others "bad" does not explain these distinctions or categorize anything usefully in this discussion.

your life is unlike Rand's or Roark's. yours is worse. calling it "good" doesn't address that. calling it "bad" doesn't help either.

and it's unclear to me what problem is to be solved by giving a single overall characterization of your whole life.

the attribute "has some significant stuff to lose like..." is more meaningful. i see the actual meaning, and relevance, of that attribute.


curi at 5:49 PM on January 30, 2017 | #8315 | reply | quote

>> You're giving up?

> No. I said in #8307:

>> I think the discussion is going in circles and I'm not sure how to resolve that.

>I still don't think I've resolved it, so I'm trying other approaches.

The circle I see is:

You say you have a good life

Someone points out a flaw

You deflect or ignore

Repeat

(you haven't done that always, but it seems like it happens a significant amount of the time)

Whether you actually have a good life is irrelevant.

What matters is: are the criticisms of your life true or not?

If they're accurate, no matter whether your life is "good" or not now, your life can be *better*.

Here's my overall sense for your position:

You think the social in your life that you need to maintain your financial position is an acceptable cost. You don't think the consequences cost you more than the profit you make.

Something like that?

Or is it that you don't think it costs you and are unconvinced that it is bad?

Do you think of social stuff as just part of work? Like it's not really different from other things you can get paid for? It's a thing you know how to do and can increase your profit by doing?


Anonymous at 10:22 AM on January 31, 2017 | #8317 | reply | quote

> You think the social in your life that you need to maintain your financial position is an acceptable cost. You don't think the consequences cost you more than the profit you make.

i don't think that's an accurate statement about AnonITA. I think he doesn't understand much about the cost of the social. I think he's aware FI says social is bad, and he agrees in some specific cases (like Kim Kardashian) but he doesn't know much about what's wrong with social in a lot of other situations that come up in his life. he also knows smalltalk is bad, i bet, but only mildly -- i'd guess he sees that as minor (e.g. it's kinda boring).


curi at 7:08 PM on January 31, 2017 | #8318 | reply | quote

> I am trying to think about "doesn't require being social" to a high standard, which I assume you were too.

I don't consider a UPS driver job to be social to the standard of it fucks up your life. I think you can be pretty damn non-social, successfully deliver packages for UPS, and have a decent life. This is despite the fact that UPS drivers occasionally speak to customers and coworkers.

By contrast, a bartender job is social in a life-ruining kind of way. It's incompatible with a decent life. Kim Kardashian's job is also life-ruining. Sales jobs where you interact directly with customers you're selling to are generally awful. (Marketing where you e.g. design ads and only indirectly deal with customers is often awful too, but it has much more possibility of being OK than interacting directly with customers doing sales or e.g. Twitter marketing.)

There are lots of jobs that don't ruin an "I don't want to be social project" and are compatible with having a decent life. And lots of jobs are no good.

You shouldn't seek a "high standard" for no reason. You should seek a *useful standard*. Is there a problem here, or not? The standard should be problem-based, not just looking for arbitrarily high precision or high target amounts for no reason.

There are a lot of jobs where it's harder to evaluate if they're OK or not. And it also depends on your attitude to the job. The same job might be OK if you don't seek promotion and don't expect to keep the job for more than 5 years, but involve a bunch of social internal politics if you approach it as a career and try to get promoted eventually into upper management.

Roark was able to work at Keating's architecture firm. Not a big deal. He did get fired after a while though because he didn't want to do things that would have been bad for him. Did that have to happen? Nah. With a bit more luck he might have stayed there for 10 years without running into some nasty conflict. Lots of employers are pretty forgiving of various stuff if you aren't directly annoying customers and you are doing good work that makes them money.


curi at 11:46 PM on January 31, 2017 | #8320 | reply | quote

> In that context, I think "doesn't require being social" to a high standard includes meeting all of:

> - Doesn't require physical presence or "visibility" (video or pictures) beyond the actual, objective requirements of the job if any.

You should try out the Howard Roark test. Would Howard Roark do it? Would it be a big deal or problem for him?

You can do a similar test with Rand, Galt, etc

When Roark worked at offices, including Cameron's, was he required to be there in person more than objectively necessary? Yes. He could have done some more of his work from home.

But did that ruin the jobs for him? No. It was alright. It had some practical downside, but it wasn't compromising Roark's values to show up in person to do work.

Also Roark would more or less not care if people saw him. And why should he care? It'd be bad if he was dressing to impress and playing appearance games. But he wasn't. It's one thing for a job to ask you to come in in-person. It's another for them to have some objective, clear, easy-to-follow dress code which is inconvenient, and it's another to partake in competition to dress more impressively and gain social status. It's when people are putting their creativity into dressing to please others that it starts ruining lives.


curi at 11:51 PM on January 31, 2017 | #8321 | reply | quote

> - Doesn't require producing things for solely or mostly social reasons, like pretty reports or proposals or other busywork. "Just the facts" kind of reports on objectively valuable data are the only kind ever required.

whether this sucks to do depends on what it takes to do it.

are you competing with another guy to give a flashier presentation to get a promotion? ok you're playing social games and it's awful.

but are you simply required to upload your reports to a piece of software which automatically makes them pretty? so e.g. you write in markdown and then your boss reads it in a pretty epub or pdf? that's not going to fuck you over. that's fine. understanding what people consider pretty formatting isn't your problem so you aren't having to learn nuances of pleasing others and understanding what they want. you just let the software handle things (and behind it there's some designer who understands current taste and stuff, and his job partly sucks, though partly there's also objective standards for formatting), no big deal for you.


curi at 11:55 PM on January 31, 2017 | #8322 | reply | quote

> What is the difference between "life-ruining" social and not-life-ruining social?

the only way to understand this is to learn about what's bad about social. you need a conceptual understanding. then you can evaluate which stuff is awful and which isn't.

you can't learn this from a quick paragraph or list.


curi at 11:59 PM on January 31, 2017 | #8323 | reply | quote

> That (social) problem doesn't bother me much on its own

you know little about social.

basically what's at stake here is a life devoted to static memes (as much of your life is), or not.


curi at 12:01 AM on February 1, 2017 | #8324 | reply | quote

> Do you have any suggestions for how to proceed from here?

actively seek to be/become more of a thinker.


curi at 12:03 AM on February 1, 2017 | #8325 | reply | quote

>he also knows smalltalk is bad, i bet, but only mildly -- i'd guess he sees that as minor (e.g. it's kinda boring).

Why is smalltalk majorly bad?


Anonymous at 6:14 AM on February 1, 2017 | #8326 | reply | quote

> Why is smalltalk majorly bad?

i don't know a short answer to this that would work for a typical person in our culture.


Anonymous at 6:55 AM on February 1, 2017 | #8327 | reply | quote

>> You think the social in your life that you need to maintain your financial position is an acceptable cost. You don't think the consequences cost you more than the profit you make.

>i don't think that's an accurate statement about AnonITA. I think he doesn't understand much about the cost of the social. I think he's aware FI says social is bad, and he agrees in some specific cases (like Kim Kardashian) but he doesn't know much about what's wrong with social in a lot of other situations that come up in his life. he also knows smalltalk is bad, i bet, but only mildly -- i'd guess he sees that as minor (e.g. it's kinda boring).

I agree with your assessment of AnonITA.

I'm not sure how it's incompatible with my statement though.

Is it that I didn't include comment that his judgement of the cost of social in his life is an underestimate? Was it implied that his judgement is knowledgeable and accurate?


SN at 9:00 AM on February 1, 2017 | #8329 | reply | quote

#8325

> actively seek to be/become more of a thinker.

Suppose I agree this would be a good thing. What concrete steps would I take?


AnonITA at 3:19 PM on February 4, 2017 | #8352 | reply | quote

there are many options for proceeding and you know lots of them.

one option is to read and discuss philosophy books.

if you don't like it and some others, there's always the option of discussing options. you could post some options you're aware of and what you don't like about them, and ask about other options you don't know about and ways to modify options you do know about to fix your complaints.

i don't want to give you an introductory list of options here because i think it'd be a waste of time since i think you already know some and it'd be better to move on to the next step of the discussion.


curi at 5:42 PM on February 4, 2017 | #8356 | reply | quote

quote from #8314:

>Rand's life, Trump's life, Hitler's life, my life, and a crack whore's life are (or were) all problematic. True, but misses the point. Some people have a lot more to lose than others. They have a lot more to lose because they've done more good things in their lives than people with less to lose.

>I think having more to lose matters to what problems they should work on solving and how they should go about that work in terms of things like risk and starting over.

you need to keep making tons of OPEN ENDED progress or u r dead. so having some good things going isn't an argument against making open-ended progress...

actually, having a lot to lose makes it MORE URGENT that you make open-ended progress, if u look at it a certain way...

Having a lot to lose means you've got some good stuff going on already. Which means that what you could reasonably accomplish in your life is more than someone with way less good stuff going on -- they'd have to roughly get to your level first before considering better stuff, and that could take lots of time.

But that means your near-term opportunity cost of doing stuff besides making progress is way higher than theirs.

Like if a loser welfare recipient learns enough good values to become self-supporting, that is great progress, good for him. If he doesn't he just stays a welfare recipient, and that sucks. But it doesn't affect stuff a bunch if you have one less welfare recipient and one more low wage worker. He'll need to make more progress than that if he really wants to have a good life (and not just a not-super-awful life)

OTOH if a person who isn't a loser fails to make progress, they could be giving up the possibility of actually doing something really interesting and worthwhile in the fairly near term!

Or like, thinking of civilizations ... if you are some shitty primitive tribe, and you don't make progress by e.g. instituting liberal ideas, that sucks, you're missing out, but basically its just status quo. you're already pretty close to the bottom rung of what's possible in terms of human existence, and you're staying there.

OTOH if Western civilization doesn't make progress and learn how to defend itself against leftism and Islam, we'll be literally giving up the somewhat near-term possibility of SENS and nanotech constructors and Mars colonies, and plunging into a new totalitarian dark age. Sad!


Mysterious person at 8:01 AM on February 5, 2017 | #8375 | reply | quote

great point.

more to lose = more to lose IF YOU DON'T MAKE PROGRESS.

you automatically lose if you don't make progress because problems are inevitable so you always need rapid progress or you're fucked. so if you have a lot to lose, that's incentive to make progress!!!!


Anonymous at 8:45 AM on February 5, 2017 | #8376 | reply | quote

#8356

I think I need to start by discussing whether to rule out options that involve intentional self-coercion.

From a discussion I'm not involved in: http://curi.us/1950-open-discussion#c8387

> nobody's discovered a reliably pain-free way to go from an irrational mentality to a rational mentality.

> [...] you don't need to always take your emotions so seriously or let them completely control your life. you can try disrespecting them a bit while you're trying to learn and improve, and see what happens.

My current problem with FI is not "pain" per se. I don't find the posting I do here, or on FI list, painful. If nothing else, it's a fun & interesting break from other stuff I do. I think it's more than that too - I get good insights sometimes, learn helpful stuff, etc. But point is, I'm not currently making myself do anything here that I don't want to / feel like doing.

But as has been observed in this thread, what I do on FI isn't really serious either. I think taking FI seriously would be painful/coercive.

Like when someone recommends reading and discussing philosophy books. At any given time I've tried that, I don't want to / feel like doing it. So I don't.

Or when someone recommends pursuing a few threads to completion rather than letting them drop. I don't want to / feel like doing that. So I don't.

etc.

I know FI claims that by taking FI seriously I could change how I feel about doing stuff like the actions FI recommends for learning FI.

But it's kind of a chicken-and-egg problem: how do I seriously learn FI if I never want to / feel like doing the things that it takes to seriously learn FI, and FI says not to coerce yourself?

BTW, I don't think I avoid intentional self-coercion in any other area of my life that I take seriously.

It's not like all of my days are filled with intentional self-coercion. Far from it. But if I judge that something really needs to get done in some non-FI area of my life and I don't want to do it and I can't find another way to deal with it fairly quickly...oh well. I just make myself do it anyway. And usually afterwards I'm glad that I did, and things turn out better for me than if I hadn't.

One of the (not fully developed) criticisms I have of FI's coercion concept is that it seems to me the more successful people in the world have the right mix of BOTH doing stuff they really love to do AND making themselves do some things they don't want to but are nevertheless important to their success. I realize FI disagrees with that view, which is one reason why I've exempted FI from deliberate self coercion.

Anyway, my question is:

Should I continue to keep intentional self-coercion off the table with regard to learning FI? Or should I consider options that include making myself do some things that I might not want to / feel like doing at the time?


AnonITA at 4:28 PM on February 5, 2017 | #8389 | reply | quote

>It's not like all of my days are filled with intentional self-coercion. Far from it. But if I judge that something really needs to get done in some non-FI area of my life and I don't want to do it and I can't find another way to deal with it fairly quickly...oh well. I just make myself do it anyway.

Can you give some examples of when you've done this?


Anonymous at 7:34 AM on February 6, 2017 | #8407 | reply | quote

Sure.

I don't like getting shots. But I need to get routine vaccinations like flu shots. If I had waited until I felt OK with getting shots I'd have not done it in prior years and risked getting the flu, tetanus, etc. Instead I just make myself get the shots when it's time for them.

I don't like re-doing status reports. Sometimes a big-wig at work wants a status report in a particular place/format. If after doing the socially calibrated level of pushback I'm allowed to do, sometimes they still want it. I don't want to do it, but I make myself do it anyway so I can keep getting paid.

Another work example is there's this yearly "class" HR requires in what I guess is fairly typical corporate PC bullshit. Don't harass your co-workers or do unethical shit blah blah blah. It's boring, I already know it, don't want to do the class. But I make myself do it anyway every year so I can keep getting paid.

I don't like doing taxes. I've hired the help it makes sense to hire (as an aside, too much / the wrong kind of help in this area can expose a person to unnecessary financial risk). I never feel like doing taxes, but I make myself do what I need to in order to neither pay a bunch extra to the government I don't have to, or be at risk for fines or tax evasion.

I don't like looking for things. I have processes and routines that mostly avoid the need to look for things. But sometimes those don't work. if I know I have something, don't know where it is and I need it and can't easily get another one, gotta search. I make myself do the search even though I don't like doing it.

Caveat: these are all examples that people might have suggestions on how to solve without self-coercion. Maybe some of the suggestions I haven't already thought of, but lots I probably already have. I'm not describing everything I have thought of or tried before self-coercion as that's not the purpose here.


AnonITA at 1:25 PM on February 6, 2017 | #8417 | reply | quote

#8417:

>I don't like getting shots. But I need to get routine vaccinations like flu shots. If I had waited until I felt OK with getting shots I'd have not done it in prior years and risked getting the flu, tetanus, etc. Instead I just make myself get the shots when it's time for them.

what's the issue here, exactly?

last time i got a flu shot i got it at the supermarket. so i went to the pharmacy counter and got a shot and then did some shopping and stuff. i was totally 100% fine with it. it was quick and convenient. and it's valuable cuz it reduces the chance of getting sick. and its not expensive. what's not to like?


Anonymous at 6:27 PM on February 6, 2017 | #8418 | reply | quote

> But it's kind of a chicken-and-egg problem

yeah. and no general-case, straightforward solution to this is known.

for any given situation, there's a way to make progress. but it depends on lots of details of your situation like what resources you have and your preferences. and i don't know a ton about those. so i can't tell you what to do very well b/c there isn't just a generic answer and i don't know tons about you.

this is different than lots of other issues. should you hit your kid? NO. i don't need to know a bunch of details about your life to answer that one. i've got a one-size-fits-all answer which is easier to share with people.

> FI says not to coerce yourself?

FI doesn't say that. FI says there are always better options.

And FI says there's a way to always find better (non-coercive) options fast enough which people can learn and it's pretty small/simple, rather than something that'd take decades to learn. And it works as long as people use it rather than e.g. getting caught up in their irrationalities and doing dumb shit.

This is a big deal because it's fast enough and cheap enough in terms of resource use. The claim "Problems are always solvable given unlimited time and resources." is pretty easy for many people to accept and not that big a deal. It's easy to imagine if you had a few decades and a billion dollars to figure something out, there'd be a solution.

But there's a method for getting non-coercive solutions one can use in their life instead of only hypothetical solutions given more time/resources. (These are solutions to the standard of non-coercion, they don't solve everything. E.g. it doesn't mean you can build a nuclear fusion power plant cheaply and quickly. But you can cheaply and quickly *not be coerced* over your inability to build it.)

How cheaply and quickly does this stuff work? Enough to not be coerced. Enough it's useable in your life. And the method itself is pretty small and learnable, not something that'd take 500 years of study.

But anyway, what do you do if you don't yet know this method? Learn it. And if you find that coercive? Well that sucks. I dunno. FI doesn't have some clear statement about what to do about that difficult situation. It does not say "whatever you do, don't coerce yourself into learning how to deal with coercion." It says more like "good luck" and also e.g. "try to figure out what resources, abilities, etc, you DO have in your life and make progress with those".

And what do you do if you know the method but sometimes don't use it? Try to learn. But there's no super clear answer. The method doesn't guarantee you use it. It's a tool you can use. But for various reasons people often sabotage stuff, make bad choices, etc. What's needed is progress, self-improvement, etc. And in the meantime, uhh, things are tough. So it's really urgent to improve!

> Should I continue to keep intentional self-coercion off the table with regard to learning FI?

that's really up to you. but i don't know why you heard "there is a better way" as "never do your current way even before you know a better way, but only apply this limitation to learning FI stuff".


curi at 8:40 PM on February 6, 2017 | #8421 | reply | quote

@#8417 it's sad how mundane and basic the problems people get stuck on for their whole lives are. they often can't even solve lots of stuff like this, let alone more interesting or difficult stuff.


curi at 8:42 PM on February 6, 2017 | #8422 | reply | quote

> But there's a method for getting non-coercive solutions one can use in their life instead of only hypothetical solutions given more time/resources.

I assume the method you're referring to is the one described at:

http://fallibleideas.com/avoiding-coercion

If you meant something else, please let me know what you meant.

I won't claim a philosophically serious knowledge about the method I linked to. Not the least because I don't know what would actually meet that standard.

But I'm not unfamiliar with that method. I think I know enough to talk about it and to make a guess about why I'm not currently able to use it effectively in some areas of my life.

Since someone asked for more information about the shots example I'll use that one. What I don't like about shots is anticipating the shot and the shot itself. I have an emotional reaction that's best described as a combination of fear + disgust, at the thought or experience of a needle piercing my skin. This is just about the needle. It's not about what's going in to me (vaccines don't bother me as such, and I've taken i.e. some specialized oral vaccines for some stuff before traveling with no unwanted emotions). Nor is it about the possibility of blood (seeing blood, even my own, doesn't bother me). But even like stick pins in clothes or sewing needles do bother me in approximately the same way as flu shots. I have trouble trying on new shirts etc. until I make sure there are absolutely no pins in them. And BTW I've also looked into nasal flu vaccines and they haven't been available to me.

Anyway, I don't like/want those emotions about needles and I don't consider them to be the correct response to getting a shot of a vaccine against an unwanted disease.

But I have the emotions anyway, and don't know how not to have them. I have read http://fallibleideas.com/emotions but haven't made progress on these emotions. Which is a topic in itself, but back to the method.

P1 = Should I get flu shots each year?

My explicit ideas say get flu shots. I have strong criticisms of not getting a flu shot like getting the flu is a big productivity hit, it sucks, and could even lead to more serious complications or death. And I know flu shots aren't 100% effective but they're effective enough that getting one is better than not getting it.

My emotions say don't ever get any shot at all, including flu shots.

P2 = Given my explicit ideas about the flu shot and my emotions about shots disagree, what should I do?

If it's not yet the right time to get a flu shot, continue to think about it and work on the emotions until it's actually time to get the shot. OK fine, but as I said I hasn't been effective on the emotions. And time does eventually run out.

P3 = Given my explicit ideas about the flu shot right now and my emotions about getting the shot right now disagree and it's now the right time to get a flu shot if I'm going to get one, what should I do?

Explicit ideas say do it; emotions say delay further. Don't know any way to make the problem easier (I can't think of any effective P4). I either get it now, or I don't(1). So I just self-coerce, tell my emotions to STFU, get the shot, and once it's done I'm glad I did.

(1) "Now" is contextual. Sure, I could get the shot tomorrow instead of today. But ultimately too many tomorrows adds up to not getting it at the right time.


AnonITA at 5:16 PM on February 7, 2017 | #8427 | reply | quote

here's how #8427 reads to me:

first of all, assume that I'm bad at introspection, thinking, and solving problems. ok now that we have that all set, the “avoiding coercion” method doesn’t seem to work for me.

like in this example with the flu shot, i'm left facing a choice to either self-coerce and get the shot or self-coerce and not get the shot.


Kate at 8:03 PM on February 7, 2017 | #8428 | reply | quote

@#8428

It's: Given I'm *irrational* about X -- that is, not using rational methods like the avoiding coercion method to think about it -- then the method doesn't work for me about X.

It doesn't work when you don't use it. Whether you genuinely want to solve problems, or prioritize other things like being emotional, is a separate matter than the complexity and efficacy of this problem solving method. No method works when it isn't used.

Emotions are ideas. The "omg needles noooooooo" emotion is not participating in the avoiding coercion method. It's reaching conclusions in a different way. His ideas -- his thinking process -- isn't following the method.

He's clinging to non-negotiables on conflicting sides of an issue. One doesn't have to do that and should not do that.

Broadly, it takes a rational attitude to life to solve problems. Not e.g. being 100% non-negotiably in the thrall of arbitrary nonsense like "no needles because [no reason]".

Why not start solving problems in the 99% of life he doesn't have super-entrenched terrible ideas about? Start with the easy cases. Or does he have terrible ideas about most of life and no clue how to change or improve them?


Anonymous at 8:28 PM on February 7, 2017 | #8429 | reply | quote

#8427 and #8428

I can't talk about everything at once. I don't think it makes sense to talk about both the avoiding coercion topic and the emotion topic at the same time. So ya, for purposes of the discussion I was taking the emotions as a given in order to talk about the avoiding coercion method. In doing so I thought I was relying on the instructions in the method itself.

From the avoiding coercion method:

> The method of constructing P2 goes like this: given we disagree about P1, what should we do? P3 is constructed like this: given we disagree about P1 and P2, what should we do?

Seems the whole point of the method is to take some disagreement as given. Not given forever, but given for now (which is what I was talking about). So it seemed entirely reasonable according to the method to take the disagreement between my emotions about needles and my explicit ideas about needles as given.

It's possible I'm misunderstanding the use of "given" in the avoiding coercion method. In that case please explain it to me. I didn't claim to be an expert.

> Why not start solving problems in the 99% of life he doesn't have super-entrenched terrible ideas about?

Why do you think I'm not?

I expect the return on discussing the most problematic areas of life to be higher than the return on discussing less problematic areas of life.


AnonITA at 9:19 AM on February 8, 2017 | #8430 | reply | quote

> Why do you think I'm not?

well you picked the needle example.

> I expect the return on discussing the most problematic areas of life to be higher than the return on discussing less problematic areas of life.

wait, so first you deny it, then you give a reason you're doing it on purpose?

anyway your emotions aren't a disagreement and aren't participating in the avoiding coercion method. they are not playing the role of a delegation at a mediation which has your best interests as a person at heart. they aren't willing to try to figure out the best thing for you. they aren't seeking solutions.

if you don't do the method -- if you don't seek solutions -- it won't work.

what to do about your unwillingness to do the method is a tricky topic. the solution is not to start stipulating that "given one part of me is active and rejects problem solving, avoiding coercion, reason, etc..." and think you're still doing the method. you have to have some part of you that is willing to do the method active and in control to do the method.

the problem you've brought up here isn't really about needles, it's about having ideas along these lines: "i want X bad thing and i won't negotiate this demand in any way. solve that!" and then you think you're in some way pointing out some flaw or limit or difficulty with rational problem solving.


Anonymous at 9:58 AM on February 8, 2017 | #8431 | reply | quote

> wait, so first you deny it, then you give a reason you're doing it on purpose?

I think "it" above out to relate to different parts my statements, whereas you're treating "it" as a single thing.

My first statement ("Why do you think I'm not?") refers to what I'm *doing* in my life. I am solving lots of problems in my life. I deny that I'm not solving problems in the 99% of life where I don't have super-entrenched terrible ideas about.

My second statement refers to what I'm *discussing*. I am discussing the most problematic areas. And yes, I'm discussing those areas on purpose.


AnonITA at 10:15 AM on February 8, 2017 | #8432 | reply | quote

you should discuss stuff you can deal with better to try to learn about the problem solving method, instead of the stuff you're most stuck and stupid about. don't try to do 2 things at once (learn about method, deal with needles which you're especially terrible at).


Anonymous at 10:20 AM on February 8, 2017 | #8433 | reply | quote

> anyway your emotions aren't a disagreement and aren't participating in the avoiding coercion method. they are not playing the role of a delegation at a mediation which has your best interests as a person at heart. they aren't willing to try to figure out the best thing for you. they aren't seeking solutions.

Re-reading the method I think what you're trying to say is that one of the pre-requisites of the method is not being met. From the method:

> We're only going to discuss avoiding coercion when both people want to avoid it

And in the case of the needle, the anti-needle emotions represent one of the people. And those emotions don't want to avoid coercion. So the pre-requisites aren't met.

If that's what you're saying, then yes I agree.


AnonITA at 10:26 AM on February 8, 2017 | #8434 | reply | quote

> you should discuss stuff you can deal with better to try to learn about the problem solving method, instead of the stuff you're most stuck and stupid about.

I don't know how to apply this advice.

At a high level, there's roughly:

(1) problems I know how to solve and solve without self-coercion, and do in fact solve

(2) problems I don't know how to solve without self-coercion

I was asked for examples of the latter, so I gave 5 examples.

Then I was asked for more detail about one of the examples (shots), and also the method of avoiding coercion was brought up.

I thought it'd be easiest to discuss self-coercion in the particular example I was asked for more detail about. That's why I picked that example to discuss (and said so explicitly).

So, are you advising:

- That I should discuss only problems I already know how to solve without self-coercion?

or

- That I should discuss problems I don't know how to solve without self-coercion, but not one of the examples I gave?

or

- That I should discuss one of the other examples?

or

- Something else I'm not getting?


AnonITA at 11:28 AM on February 8, 2017 | #8435 | reply | quote

Circling back to #8421

> How cheaply and quickly does this stuff work? Enough to not be coerced. Enough it's useable in your life.

It seems like this needs a caveat: The method works cheaply and quickly enough not to be coerced unless one of the sources of disagreement is emotions you have super-entrenched terrible ideas about.


AnonITA at 8:10 AM on February 9, 2017 | #8436 | reply | quote

#8421 you have not learned the method and do not use it. so, no, your caveats are wrong. i also don't think you understand the meaning of the 3 sentences you quote there.

i can't tell you solutions to your problem with needles, and various others, because you have not provided adequate information for me to figure everything out for you. (note it's very hard to provide adequate information about some problems.)

and what you presented about the needle problem is only loosely connected to your real life situation and basically designed to be insoluble as presented. your question really boils down to "what if i have 2 contradictory non-negotiable preferences. how do i solve that?" and the answer is more or less that solutions are only available to people who prefer problem solving to being stuck. if you're not willing to change your mind about anything then there are no answers. if you want to engage in problem solving you could learn to do that. if you choose to be stuck you can do that, and it's not fault of any method that you're choosing to stick to some bad ideas. you also deny responsibility for this choice which also isn't helping solve anything.

and you don't know how to create your own solution, and don't seem interested in learning the method including background knowledge, and don't seem interested in trying the method on stuff you're less stuck on. that's your choice. i think it's a bad one. anyway it doesn't add caveats to ideas.

you could adopt an attitude of patient tolerance for your bad ideas as you work through them (you won't fix them all overnight) and in the big picture being happy that you're making progress even though not everything is ideal in the short term. that'd be an example of a reasonable thing to do. but the premise of your question is more or less what if you're being unreasonable? and the answer is there is a rational problem-solving method which could help you, *if* you chose to be reasonable and use it. it is not a method of making you be reasonable if you don't choose to be though -- you have to want to solve your problems and choose to use the method, it doesn't make you use it, it doesn't somehow control you to prevent self-sabotage, self-destruction, etc.


curi at 8:23 AM on February 9, 2017 | #8437 | reply | quote

> you have not learned the method and do not use it.

Right. What I said was "I'm not unfamiliar with that method. I think I know enough to talk about it and to make a guess about why I'm not currently able to use it effectively in some areas of my life."

> and what you presented about the needle problem is only loosely connected to your real life situation and basically designed to be insoluble as presented.

It's insoluble as presented because I know of no other way to think about it and present it.

If I knew of a way to think about it and present it that was soluble, my guess is I would've solved it already. Like the hundreds of other problems in life that I am successful in solving.

> [...] and don't seem interested in trying the method on stuff you're less stuck on.

I don't understand / can't apply the concept "less stuck".

The examples I listed were all situations I'd categorize as "stuck". There are different reasons for being stuck regarding boring HR classes than regarding needles. But I wouldn't consider either to be more or less stuck than the other.

There are other problems that I routinely solve or at least make progress on that I judge acceptable. I'd call those cases "not stuck".

I literally cannot think of anything to try the method on where I think I'm "less stuck". It's possible that the reason I can't think of anything because of lack of interest. But that's hard to judge.

> you could adopt an attitude of patient tolerance for your bad ideas as you work through them [...]

What does that look like in practice?

I think I know what the opposite of "patient tolerance" looks like in practice. With the needles example, it would mean go to an acupuncturist or donate blood or whatever. Deliberately seek anxiety-provoking exposure to as many needles as possible to habituate myself to them, so that flu shots cease to provoke an emotional response. Psychiatry routinely suggests that kind of shit as the "solution". I have rejected that approach.

But how would I implement "patient tolerance"?

> but the premise of your question is more or less what if you're being unreasonable?

True, but I think less helpful than a more specific characterization of my premise:

what if part of you is being unreasonable and you don't know how to change or deal with that part?


AnonITA at 11:56 AM on February 9, 2017 | #8438 | reply | quote

> I won't claim a philosophically serious knowledge about the method I linked to. Not the least because I don't know what would actually meet that standard.

>

> But I'm not unfamiliar with that method. I think I know enough to talk about it

You overreached here in what you could say about it.

> > you have not learned the method and do not use it.

>

> Right.

The answer "Right." is a very bad answer.

I'm arguing with you. You are trying to reply by agreeing with me. You can't agree with someone who is arguing with you. Someone is wrong.

I wrote what I did for a reason, which you don't understand. You should be asking a question, not posting fake agreement that dismisses the issue and tries to close that aspect of the conversation. And in the alternative if you understood everything correctly, then I'd be wrong -- I'd be misjudging the situation, arguing a point I shouldn't -- so the answer still isn't "Right".

The reason I was talking about you not understanding the method is because I thought you didn't understand it in *relevant* ways to this conversation. You seem to have agreed with me that you don't understand it in *irrelevant* ways. Why would I bring those up!?

you also unquoted my further explanation where i say your caveats are wrong and you don't understand 3 particular sentences. how is that not clear that we disagree or there's some kind of conflict or problem or something? but you selectively quote part of it then say "right". you're evading a dispute and you aren't curious about the stuff i say you don't understand. you aren't trying to think/learn, you're trying to cling to a narrative. it's so blatant. i say you're wrong about something and don't understand something and it's a short paragraph and you selectively quote the paragraph and try, ridiculously, to agree with me and leave it at that and say nothing new. (btw when i contradict what you already said, the answer isn't to quote it back to me as if i hadn't read it. i'm disagreeing with you on purpose. it's an argument. you don't seem to get that.)

> It's insoluble as presented because I know of no other way to think about it and present it.

>

> If I knew of a way to think about it and present it that was soluble, my guess is I would've solved it already. Like the hundreds of other problems in life that I am successful in solving.

it's extremely common to have problems that are in the middle ground of being neither solved nor insoluble from one's perspective. both problems you're working on and also problems you've heard of but aren't working on.

problems you've already solved were in this state previously before you solved them.

i think you believe a cure for cancer is possible. and a solution for aging is possible. those problems are neither insoluble nor solved. i'm confident if you actually thought about it you could easily list dozens more problems which are in this middle ground category you're denying and don't want to discuss problems from.

---

less stuck means e.g. you haven't just run straight into a wall and see zero options, you have various open leads to try (but it's still not going very well and you still aren't confident these leads will work).

also less stuck means you're putting less effort into staying stuck. there are some problems where, when you deal with that area, you put 99% of your creativity, effort, etc, into preventing a solution. then it's really hard to make progress. you're really stuck. but there are other areas where the figure is only 50% or 20% so you're less stuck, it's much more plausible to make progress.

---

> What does that [patient tolerance] look like in practice?

you shouldn't have written "that", it makes your question unquotable/answerable on its own. and as usual it's not even very clear what specifically "that" is meant to refer to.

an example is you fear spiders. while working on this problem you avoid visiting the spider exhibit at the zoo.

similarly if needles are that big a deal to you you could just not get a flu shot one year. that isn't really that costly and would provide you enough time to solve the problem before the next flu season if you were actually doing problem solving. (this patient tolerance thing works well when paired with problem solving, is not a recommendation for what to do with lifelong unsolved problems)

---

> what if part of you is being unreasonable and you don't know how to change or deal with that part?

you mean: what if YOU ARE UNREASONABLE and you don't know how to stop being so fucking irrational?

step 1, stop making excuses and downplaying it and trying to disown it.

step 2, learn philosophy and use it.

doesn't have to be done in this order.

---

note most of this is about AnonITA discussing badly. he's sidetracking the discussion instead of getting things focused on learning.


curi at 12:03 AM on February 11, 2017 | #8443 | reply | quote

part of the confusion and issue is this:

it takes like X effort to learn the methods of rational problem solving, avoiding coercion, etc

X is small, finite. let's say, 500.

and AnonITA's questions are more or less: what if my starting point isn't 50, or even 0, but -100 or worse? what if my starting point is -9999999? -999999999999999999999999999999? then X is really far off!

yeah if you start at negative a million, then it'd be a long road to 500. this doesn't make 500 big or hard. it doesn't change the nature or difficulty of the problem solving method. it has the inherent properties i talk about. that people destroy their children's minds -- so that easy stuff is kinda beyond them -- doesn't change the objective nature of the method.

i bet you're objecting or doubting or something but you will accept the same point with a different example.

in math, fractions aren't that hard. it'd be hard to invent them in the first place. but they just aren't very complicated to learn now that they are already well understood. it's not very hard to do things like convert fractions to decimals and back, reduce fractions, multiply two fractions, add two fractions, etc.

many people find this hard, get stuck, suck at it for life, etc. that experience does not make basic fractions math objectively difficult.

and btw if you don't know what a number is, it'd take a while to get to understanding fractions. same as if you don't know a language yet, it'll take a while to get to understanding the avoiding coercion method.

but if you have some reasonable background knowledge -- know what a number is, know what addition is, etc -- then it's just not a big deal to learn basic stuff about fractions.

this does not match everyone's experience. the majority of people try to learn fractions at school and have a rough time.

this is not because fractions are actually hard. it's because they're deeply irrational about math and also school is an awful, unhelpful place for learning. neither of those things -- the prior destruction of their mind regarding math (and much more), and the low quality of school education -- makes fractions inherently hard to understand.

AnonITA is mixing up his own status as an intellectual cripple with stuff being hard.

it's important to understand the situation for a variety of reasons including:

-- actually accepting this and taking it seriously could lead to some motivation to fix it instead of thinking one's life is pretty good

-- it totally changes where you direct effort if you want to learn avoiding coercion. most effort has to go to fixing your broken shit instead of the avoiding coercion topic itself.

-- the same ways you're broken and put the majority of your effort into preventing progress when it comes to learning about avoiding coercion, you also do on many other topics.

-- knowing what the problem situation is helps you judge potential courses of action better.


curi at 12:14 AM on February 11, 2017 | #8444 | reply | quote

i think AnonITA dramatically overestimates the completeness of the avoiding coercion FI essay. he doesn't understand that, like more or less all essays, it's a bunch of clues, not a direct transfer of solutions into your brain. it's dehydrated and you have to add water (creative thinking -- which is actually way harder to add than water). people totally misunderstand what learning is like in general because they are used to learning stuff with a TON of cultural support and also doing most of their learning as a kid and not remembering it.

and AnonITA doesn't understand that you have to have various background knowledge to understand or use the essay, like certain understanding about reason and problem solving, which he doesn't have.

understanding avoiding coercion is more like 50 books worth of total content, not 1 essay. (all the stuff on the FI site adds up to like 2 books equivalent.) and note one has to actually learn stuff, not just read through it. and 50 books is still rather small. though it's kinda daunting for people who can't read 1 book b/c they put 90+% of their intellectual effort into preventing progress instead of making progress.


curi at 12:24 AM on February 11, 2017 | #8445 | reply | quote

At first I read most of #8443, #8444, #8445 as curi raging out at AnonITA. Because out the caps of rage and swearing and harshness and length.

But I was just listening to one of curi's videos, and couldn't imagine curi raging out over this. So thought there had to be a better explanation.

So then I thought things like the capslock and swearing are *emphasis*, not anger. Because sometimes when it's really hard to get through to someone, and you explain yourself as well as you can, and it's not getting through or even getting a serious response, there's not much you can do beside add emphasis and be really blunt to avoid any room for evasion.

#8445

So wouldn't it be worth including suggested pre-reading to essays then? Like suggest books or other essays that might be worth reading first?

How often do you get questions about essays that you answer with a book suggestion? Often enough that it's worth adding the suggestions direct to the essays?

btw it might also be worth changing the "Fallible Ideas" link to eg something like "Fallible Ideas essays"

without pre-existing knowledge of the Fallible Ideas site, the name doesn't have much meaning and I don't think would get clicked out of anything but idle curiosity, adding "essays" adds content meaning even to someone who doesn't know the site


SN at 8:27 AM on February 11, 2017 | #8446 | reply | quote

#8446 there's *one* caps phrase in a ton of text and that's what stands out to you? it's just caps for extra emphasis. like **bold**, same thing. it needs extra strong emphasis to highlight the contradiction of his claim about *part* vs my statement which drops the "part of" qualifier/excuse.

also your interpretation of length is sooo opposite. if i don't like something that generally results in short replies. i write more if it's more interesting.

> So then I thought things like the capslock and swearing are *emphasis*, not anger. Because sometimes when it's really hard to get through to someone, and you explain yourself as well as you can, and it's not getting through or even getting a serious response, there's not much you can do beside add emphasis and be really blunt to avoid any room for evasion.

no. it's just that particular text needed strong emphasis b/c of how i was using it to contrast my perspective with the perspective AnonITA presents. need to emphasize a big difference there for clarity/meaning reasons.

> So wouldn't it be worth including suggested pre-reading to essays then? Like suggest books or other essays that might be worth reading first?

i could do that on like every essay. it's better to do it separately as its own topic instead of repeating it everywhere. and AnonITA is well aware of stuff like various books i recommend. everyone who cares already is. adding some to the essay wouldn't change that. and it's a *big* task to try to explain how to read/*learn* a book to ppl. ppl read books wrong so it doesn't work, so i can't just list a few books.

> btw it might also be worth changing the "Fallible Ideas" link to eg something like "Fallible Ideas essays"

which link?


curi at 9:15 AM on February 11, 2017 | #8450 | reply | quote

>> btw it might also be worth changing the "Fallible Ideas" link to eg something like "Fallible Ideas essays"

> which link?

side bar, between "Archives" and "More"


SN at 9:47 AM on February 11, 2017 | #8456 | reply | quote

#8450

> no. it's just that particular text needed strong emphasis b/c of how i was using it to contrast my perspective with the perspective AnonITA presents. need to emphasize a big difference there for clarity/meaning reasons.

Right, even my reinterpretation was poor. Like I was still imagining exasperation/running out of ideas. I guess cos I have the problem of exasperation/running out of ideas in discussion. But you're a ton better at it than I am, so it's unreasonable to think you'd get stuck as quickly.

>there's *one* caps phrase in a ton of text and that's what stands out to you?

Right. That's how my "interpreting things negatively" idea works. I pick something out like that, interpret it badly, then *everything* is then read in that context.

(in this case I was more skimming than carefully reading but even if I'm trying to read carefully it has this effect)

I think a common conventional idea to deal with something like this (for people who try anything at all) is to go back, catch it, try a different interpretation. I think this is useful, but I don't think it's enough on it's own to really fix the problem. It's defusing a problem after creating it, rather than working out how to avoid creating the problem.

Understanding *why* the interpretation happened in the first place and finding a solution can stop the problem recurring (so waste less life on misinterpreting and correcting). I think why people react emotionally typically gets surrounded by a lot of irrational stuff, like it being "instinct" or "innate" or something. There doesn't seem to be a lot of people who seriously try to change how they feel about something, they give up and make excuses.

So in my case, I keep trying to think of better ways of interpreting communication. Like I did above (though I was wrong about it this time). Or taking criticism as someone actually trying to *help*, like actually investing their time and energy into someone else's problems, often for *free*.

And if I'm wrong about that interpretation and they are trying to be hurtful? So what? If their criticism is true, they'll have helped me unwittingly anyway. If their criticisms are false, I might still learn about a new crit and work out how to answer it.

Side note: "creating a problem" - I think that's an important concept that I want to acknowledge. How much of my creativity goes to waste imagining problems? Some, at least. Any is too much.


SN at 1:40 AM on February 12, 2017 | #8460 | reply | quote

#8444

> most effort has to go to fixing your broken shit instead of the avoiding coercion topic itself.

#8445

> understanding avoiding coercion is more like 50 books worth of total content, not 1 essay. (all the stuff on the FI site adds up to like 2 books equivalent.) and note one has to actually learn stuff, not just read through it. and 50 books is still rather small.

#8421

> But anyway, what do you do if you don't yet know this method? Learn it. And if you find that coercive? Well that sucks. I dunno. FI doesn't have some clear statement about what to do about that difficult situation. It does not say "whatever you do, don't coerce yourself into learning how to deal with coercion." It says more like "good luck" and also e.g. "try to figure out what resources, abilities, etc, you DO have in your life and make progress with those".

There's a bunch of unaddressed topics in this thread, but I pulled these 3 quotes to focus on what I think matters most. Which is: What should I *do* next?

I break that problem in two: the logistics (my schedule of when to work on philosophy) and the content (what to do/study within the schedule I have alotted). I'll talk about logistics first.

If I'm going to be serious I think I need a schedule. My current mode of doing FI - when I have time and feel like it - does result in somewhat regular participation especially considering everything else I have going on. But it doesn't result in taking FI seriously. Kinda the opposite. I often get to FI when I'm least likely to be interested in taking things seriously.

1/2 hour in the morning would be doable on a regular basis, but more than that is just not logistically feasible without major life upheaval. I can picture ways it might become feasible to do 2-4 hours in the morning, but right now it's not.

In addition to the time in the morning, I could schedule an hour in the afternoon 3 days a week without causing major disruption. I won't be as fresh as in the morning, but it would still be more serious than what I'm doing now.

More than that would require some fairly significant upheaval. Which I can discuss if what I can do without the upheaval is too little to be worthwhile.

Do you have an opinion about whether 1/2 hour in the morning daily plus 1 hour in the afternoon 3 days a week is enough to even bother with trying? Or should I move on to considering more radical options?


AnonITA at 4:36 PM on February 12, 2017 | #8468 | reply | quote

i think 6.5 hours a week is plenty to be worthwhile. i don't understand why it wouldn't be. is there a deadline for learning a particular thing that you aren't sure if you can achieve this way? in the near future? b/c if it's in the distant future i wouldn't even worry about it, as you try this out you may end up adjusting your schedule up or down depending how well it goes, you can't predict way ahead.

it would help if you could also add in some multi-task time such as listening to an audiobook while commuting or exercising. most people already have a fair amount of underutilized time.

multitasking is more possible with more stuff than people typically realize, especially if you get skilled at it. for example, one can listen to audio books while playing many video games. and one can listen to audio books while watching a lot of video content. for example, a lot of video content can actually be watched muted with little loss, such as tons of sports or esports, which makes it easily compatible with an audio book. (btw if you can't follow a sport or esport when it's muted, and you aren't new to it, that's a sign you're watching it thoughtlessly and not learning much. apparently the announcer is guiding you and you don't know how to guide yourself or think about it on your own.)

another misconception about multitasking with audiobooks (also about speedreading) is you need 100% reading comprehension when you read a book or you aren't reading it right.

suppose you can multitask and get 70% comprehension. what's wrong with that? if the book is important just read it twice while multitasking. it's only if the book is like super super super important that you'd really want to have a very slow, maximal comprehension reading -- in which case since the book is such a big deal you'd still want, in addition to the extra careful reading, several other supplemental readings that don't have to be full comprehension! with low reading skill one might need really careful readings of more books to learn from them but that wouldn't prevent also wanting less careful supplemental readings of those books. and the optimized reading doesn't have to be the first reading. actually the optimal reading *shouldn't* be the first reading, it's easier to do a careful reading when you have a pretty good idea of what's in the book. (there is a common problem though which is people lose interest once they've read a book so they won't do a really careful reading unless it's the first reading. note this clashes with the premise this book is so important to you and you want to learn the details really well. people do it sometimes because of irrationality and sometimes because the book is less important to them than they explicitly claimed.)

if you listen to a book at 500 wpm twice (a very reasonable speed, not super fast, for a skillful reader) you can do that faster than a single really careful, slow reading *and* multitask it. this is efficient!


curi at 3:55 AM on February 13, 2017 | #8471 | reply | quote

> i think 6.5 hours a week is plenty to be worthwhile. i don't understand why it wouldn't be.

I don't know why it wouldn't be either. But I asked because it falls into a category I'll call: things I don't know enough about to make a conclusion without exposing to external criticism.

> it would help if you could also add in some multi-task time such as listening to an audiobook while commuting or exercising. most people already have a fair amount of underutilized time.

I already do a bunch of multi-task stuff, and agree most people already have a fair amount of underutilized time. I don't think I have much additional such time available. I don't expect you to take my word for it generally. Maybe I'm lying to myself. I will say that none of your specific examples apply to me. Commuting, video games, and watching sport/esport play approximately zero role in my life. I exercise, but always either with other people where it already doubles as social/family time, or where the exercise itself is a multi-task (i.e. bike ride to do errands) and I need to keep my audio channel clear for safety reasons.

I think substitution for things I'm already multitasking is a more realistic option for me than finding new multitasking time.

I listen to radio news/talk while I'm in the shower and could listen to an audiobook instead. But that's a fairly small smount of time. And I get more than zero value from the radio news/talk - some of the information I'd want to get elsewhere.

I commonly read things (including FI, but other stuff too) while my audio channel is occupied with work stuff that I'm not directly participating in but need to listen to. I could change what I read under those circumstances some. I also do suitable mundane chores with that time (cordless phone with headset, on mute) which makes those chores unavailable for other audio multitasking.

Anyway, I don't see much opportunity on the multitasking front but maybe a little.

I think the next question goes back to:

> one option is to read and discuss philosophy books.

Considering the option of reading and discussing philosophy books: if I have 1/2 hour each morning + 1 hour in the afternoon 3 days/week would it be better to:

- Aim to read no more each day than I can also post about that day (even if it's something like, "I read chapter X paragraphs Y-Z and have no thoughts/comments - what did I miss?"

- Aim to post only on the afternoon days, or weekly

- Read until I think I have something significant to post about

- Something else?

Also, any crits of starting with FoR?


AnonITA at 6:59 PM on February 13, 2017 | #8477 | reply | quote

> or where the exercise itself is a multi-task (i.e. bike ride to do errands)

the errands at the end of the bike ride have zero relevance to using an audio book while riding. that it's already one kind of multitask doesn't mean it can't be several types of multi-task at once. the errands don't interfere here.

safety is a real issue. generally when biking i set my volume low enough to hear my surroundings well. i mostly try to bike on low-traffic roads. so if i can't hear my book when a car passes me, that's alright, most of the time there isn't a car. but this doesn't work well to hear much of the book if you're in city traffic with constant cars. it's also hard to hear on steep, long downhills and i often pause for those (plus that requires focus if there are turns).

biking in traffic sucks anyway and isn't fun. i think it only makes much sense at a pretty limited set of distances. it has to be short enough not to use a car, e.g. so finding parking is too much trouble relative to the trip length (there's also the scenario of not having a car available which I'm guessing isn't your reason). and it has to be far enough not to walk. walking is more pleasant than biking with a bunch of cars and also multitasks with audiobooks great even in noisy environments.

you can also listen to audio books while doing most errands. like shopping.

---

about books, as a starting point i'd suggest reading amounts you can write about the same day. writing in the same session may be better depending how well you remember details later and can continue after an interruption.

i think FoR is a good book to start with. FoR and BoI are so so so must-read and I generally recommend reading them before any Popper. especially whole Popper books -- reading specific essays/chapters/sections of Popper works better early on instead of whole books, but requires knowing which to read.

i think you should have some comments (even if brief) very frequently while reading. if you don't, something's wrong. if you think some comments aren't worth writing down, that's fine as long as you have some you do write down for every reading session. but if you read for 30min and have nothing to say, something's wrong.

say you read 5 pages. if the book says something significant you should have a comment and/or question. if you understand it really well you should be able to add something, criticize something, talk about something related, state some further problems it leads to you don't have answers to but which would be good to figure out, or even explain why you don't think this topic is important for you to focus on (could be b/c you're more interested in something else, e.g. i think the time travel part of FoR is not important for most people. could also be because you identify this part of the book as being parochial details to set up another part of the book which is more important. if you think that, say so. you can write that in a couple sentences and that's way better than silence). and if you don't understand it really well you should have a question or try to explain it yourself and get feedback, or you could try to argue or criticize.

yes you can also ask what you missed or could have said if you have no comments.

a really frequent feedback cycle is crucial until one is very very skilled. and not just feedback from others but writing stuff down is a way to take seriously your own feedback instead of just having it really vague in your head. even if no one ever replied to you, writing down comments frequently would be beneficial.


curi at 3:18 AM on February 14, 2017 | #8479 | reply | quote

Should I have a different book for whatever audiobook listening I am able to do?

I think I should. I think it would be hard to go from reading text to audio book and back again because of stuff like finding my place in different mediums and having different retention amounts.

Any suggestion what might be a good audiobook?


AnonITA at 5:15 AM on February 14, 2017 | #8480 | reply | quote

yeah using different books is reasonable. also i think the biggest issue you want a book you care less about for the audio book. it's way easier to multitask if it's not a big deal to miss a little or not follow a little bit and not rewind.

one thing you could do is read anything you already wanted to read and work on speed.

rereads also work well b/c if you miss something you can remember it. so e.g. a Rand novel could work well.

could read softer Mises like anti-cap mentality. could read szasz or feynman. feynman is especially easy reading.


curi at 5:30 AM on February 14, 2017 | #8481 | reply | quote

> if you listen to a book at 500 wpm twice (a very reasonable speed, not super fast, for a skillful reader) you can do that faster than a single really careful, slow reading *and* multitask it. this is efficient!

How long did it take you to get comfortable listening at 500 wpm?

Was there some specific skill or technique you needed to get to that?

Is there something more to speed listening than gradually increasing speed and getting used to it?

I currently listen to stuff on Audible at 1.85x speed (or slower for more content-dense stuff) which I'd approximate to be around 250-300 wpm. I'm pretty comfortable with that. I guess I'll try boosting it a bit more. I normally listen on the way to and from work (half an hour walking each way) and sometimes at lunch.

I use a bluetooth earpiece in one ear so I can hear stuff around me with the other.


SN at 3:30 AM on February 19, 2017 | #8493 | reply | quote

> How long did it take you to get comfortable listening at 500 wpm?

i don't know, and i learned related skills first: RSVP speed reading and watching video faster (mostly TV).

I learned this stuff gradually over a few years I guess. Takes a while.

I mostly gradually increased speed. But also I sometimes set the speed higher than I'm comfortable with and try it for a bit. When going back down, it can make my old speed feel slow.


curi at 9:57 AM on February 19, 2017 | #8497 | reply | quote

Want to discuss this? Join my forum.

(Due to multi-year, sustained harassment from David Deutsch and his fans, commenting here requires an account. Accounts are not publicly available. Discussion info.)